You are on page 1of 64

NP 1 knowledge base.

This is comparable to
the ______that physicians use as their
1. It is a stimulus, either external or knowledge.
internal, that produces tension and results A. Psychological system
in a degree of instability. B. Interpersonal system
A. Equilibrium C. Dependency subsystem
B. Stressor D. Biological system
C. Subsystem
D. Homeostasis 7. It promotes helping behavior that calls
for a nurturing response. Its
2. Which subsystem has the dual consequences are approval, attention,
functions of procreation and gratification? or recognition, and physical assistance.
A Aggressive This refers to:
B. Dependency A. Attachment subsystem
C. Achievement B. Aggressive subsystem
D. Sexual C. Dependency subsystem
D. D. Ingestive and Eliminative
3. Who was the early proponent of subsystem
nursing as a science as well as an art?
A. Sister Callista Roy 8. The conservation model of Myra Levine
B. Dorothy Johnson shows that family solidarity is a way of:
C. Imogene King A. Conservation of personal integrity
D. D. Myra Levine B. Conservation of structural integrity
C. Conservation of energy
4. Who proposed that the science of D. Conservation of social integrity
nursing necessary for effective nursing
care included a synthesis of key concepts 9. Eddie enhances his self-esteem through
drawn from basic and applied sciences? good personal hygiene. He wants to
A. Sister Callista Roy conserve his:
B. Dorothy Johnson A. Personal Integrity
C. Imogene King B. Social integrity
D. Myra Levine C. Structural Integrity
D. Energy
5. The of nursing should focus on the
patient as an individual and not on the 10. She proposed the use of 4 conservation
specific disease entity: principles to guide nursing practice:
A Standards & practices A. Martha Rogers
B. Philosophy* B. Myra Lavine
C. Process & therapy C. Margaret Newman
D. Science & art D. Myra Levine

6. Johnson stated that a nurse should use 11. The Conservation Model is focused on
the behavioral system as their the
A. Prevention of the spread of disease 17. When the nurse is setting goals for
B. Promotion of individual’s wholeness the patient, what step of the nursing
C. Prevention of the spread of disease process is she already at?
D. D. All of the above A. Assessment
B. Implementation
12. Based on the ADAPTATION MODEL, C. Evaluation
the person is a/an: D. Planning
A. Biopsychosocial adaptive system
B. Biological-adaptive system 18. All but one is the classification of
C. Adaptive system STIMULI:
D. Maladaptive system A. Focal
B. Contextual
13. The following are the feedback C. Conceptual
cycles, EXCEPT? D. Residual
A. Input
B. Output 19. Which stimulus has the most
C. Throughput immediate effect on a person? A Focal
D. Parietal B. Contextual
C. Conceptual
14. Which feedback cycle pertains to the D. Residual
control processes?
A. Input 20. Environment refers to the internal
B. Output and external environments of the person.
C. Throughput Internal environment is composed of the
D. All of the above physiologic and pathophysiologic domains
of the person. External environment has
15. In the ADAPTIVE MODES three levels: perceptual, operational, and
presented by Roy, which one reflects social conceptual. Lance knows that this level
integrity and other role-related concepts? refers to things that physically affect the
A. Physiologic-Physical individual but may not be perceived by the
B. Self-concept Group identity individual himself:
C. Role Function A. Perceptual level
D Interdependence B. Operational level
C. Conceptual level
16. Which phase/step of the nursing D. Any of the above
process is involved when the nurse gathers
data about the patient's condition? 21. It is a body defense mechanism
A. Assessment that protects the body tissue from insults
B.Diagnosis in an unfriendly environment: A. Fight or
C. Planning flight*??
D. Evaluation B. Response to stress
C. Inflammatory response
D. Perceptual awareness
22. It refers to the wear and tear of body C. Martha Rogers
tissues that reflect the body's continued D. Joyce Travelbee
response and adaptation to stressful 27. Martha Roger's theory asserts that
situations. It is characterized by nurses should focus on:
irreversibility. It influences the way the A. Patient's needs
patients respond to nursing care B. Decreasing the stressors
A. Fight or flight C. Totality of the person
B.Response to stress D.Prevention of Diseases
C.Inflammatory response
DPerceptual awareness 28. Rogers viewed the person as an
open system in constant process with the
23. The primary purpose of nursing is: open system of the environment. She
A. Health promotion added that man is describe by the following
B. Disease prevention statements, except:
C. Rehabilitation A. Not unified whole possessing his own
D. Treatment integrity and manifesting characteristics
thought, sensation, and emotion, B. In
24. Are statements of clinical judgments continuously exchanging matter and energy
about the individual, family or community with the environment
response to actual or potential health C.Identified by pattern and organization and
problems and life processes that reflect reflects his innovative wholeness D.
patient behaviors patient status and provide Characterized by the capacity for
the basis for selection of nursing abstraction and imagery, language and
interventions to achieve desired outcomes: thought, sensation, and emotion.
A. Nursing process
B. Nursing diagnosis 29. It refers to the state in which the human
C. Nursing analysis being is regarded as a unified whole which
D. Nursing planning is more than and different from the sum"öf
the parts:
25. It refers to the actualization of both A Wholeness
innate and obtained human potential B.Openness
gleaned from rewarding relationships with C.Open systems
others, goal directed behavior, and expert D. Undirectionality
personal care
A. Nursing care 30. Are the fundamental unit of the
B. Optimum health living and the non-living? A Energy
C. Wellness fields
D. Optimum care B. Environmental field
C Pan dimensionality
26. Who postulated the WHOLISTIC D. Wholeness
concept that the totality is greater than the
sum of its parts? A. Sister Callista Roy 31. The person according to Neuman's
B. Virginia Henderson theory has 3 lines of defenses, which
among the following lines below serves as a
guide to assess deviations from the client's 36. On the other hand, this type of
usual wellness? prevention focuses on the return of system
A. Flexible line of defense client balance, or client wellness:
B. Lines of Resistance A Primary Prevention
C. Normal line of defense B. Secondary Prevention
D. Normal lines of resistance C. Tertiary Prevention
D. Any of the above
32. While the line that serves as a
protective layer for preventing stressors 37. The Systems Model of Betty Neuman
from breaking through the usual wellness identified stressors as capable of producing
state is? either a positive or negative effect on the
A. Flexible line of defense system. She described them as
B. Lines of Resistance interpersonal , meaning
C. Normal line of defense
D. Normal lines of resistance A. Occurring within the person
B. Occurring outside the individual
33. In the SYSTEMS MODEL, there are C. Occurring outside the individual
3 modalities of prevention. What type of D. Occurring within the environment
prevention is carried out when the client is
already manifesting the signs and 38. Natural calamities like flood &
symptoms from stress? A Primary landslides are examples of: A
Prevention Intrapersonal stressors
B. Secondary Prevention B. Interpersonal stressors
C. Tertiary Prevention C. Extra- personal stressors
D. Any of the above D. Both A and C

34. What type of prevention is 39. To maintain & retain systems


appropriate before symptoms occur? A stability, the nurse can do which of the
Primary Prevention following?
B.Secondary Prevention
C.Tertiary Prevention A. Educate the patient how to prevent
D. Any of the above dengue infestation
B. Administer the medication as
35. What is the focus of nursing in Primary ordered by the doctor
Prevention? C. Administer immunization to the
A Return of system client balance or client children
wellness. D. All of the above
B. Strengthen the flexible line of defense
C. Build up the internal lines of resistance. 40. Arrange the following steps in
D. All of the above nursing process
I. Diagnosis
II. Planning
III. Evaluation
IV. Implementation 44. Which of the following is considered
V. Ässessment as rules of performance in a redesigned
Health Care System?
A. V, i, i, iii and iv A. Safety is a system property &
B. V, i, iii, i and ii focused on increasing errors
C. V, iii, ii, iv, i B. Waste is continuously decreased
D. V, i, ii, iv and iii C. Transparency is necessary through
41. An area of nursing that involves withholding information to patients &
comforting and caring for people of all ages families.
who are dying? D. Cooperation & communication
among clinicians are not priorities
A. Promoting Health and Wellness but needed
B. Preventing illness
C. Palliative care 45. This is an organization that analyzes
D. Providing medications and evaluates current performance and
uses the results to develop focused
42. A nurse has instituted a turn improvement actions:
schedule for a patient to prevent skin
breakdown. Upon evaluation, the nurse A. Quality Improvement
finds that the patient has a stage II B. Performance Improvement
pressure ulcer on the buttocks. Which C. Quality Improvement Data
action will the nurse take next? D. Quality Data

A. Reassess the patient and situation 46. What is the meaning of the PDSA
B. Revise the turning schedule to Cycle?
increase the frequency. A. Plan, Do, Study, Assess
C. Delegate turning to the nursing B. Plan, Do, Study, Access
assistive personnel. C. Plan, Do, Study, Act
D. Apply medication to the area of skin D. Plan, Do, Study, Alter
that is broken down.
47. In providing holistic care to Tin, the
43. The nurse inserts an intravenous nurse should consider all dimensions of the
(IV) catheter using the correct technique patient's health - physically, emotionally,
and following the recommended steps psychologically. socially, and spiritually.
according to standards of care and hospital This best exemplifies what nursing role?
policy. Which type of implementation skill is A. Case Manager
the nurse using? B. Client Advocate
C. Change Agent
A. Cognitive D. Caregiver
B. Interpersonal
C. Psychomotor 48. Part of the providing services and
D. Judgmental care, Nurse Sam must speak to Clee to
relay information and/or to assess verbal
and non-verbal cues from the patient to
determine any client concerns. By so doing, nurse, you know that this is on which topic?
the nurse assumes what role? A. Prospective Payment System
A. Communicator B. Managed Care System
B. Leader C.Capitation Payment System
C. Teacher D. Pay for Performance
D. Counselor 52. Nurse Aya is providing restorative
care to a patient following an extended
49. Patient Ying is admitted in the hospitalization for influenza. Which of the
psychiatric unit of Metro Iloilo Medical following is an appropriate goal for
Center, due to alcohol addiction. Nurse restorative care?
Sam, assist him in his therapy while A. The patient will be able to heal
coordinating his plan of care to other health without signs of infection.
care team. Through this, he can modify his B. The patient will express concerns
behavior. This exemplifies what role of the related to going home.
nurse? C. The patient will identify strategies to
A. Nurse Leader boost the immune system.
B. Counselor D. The patient will be able to walk 100
C. Client Advocate feet without breath shortness.
D. Nurse Manager
53. Which of the following nursing
50. In advanced practice nursing, which activities is provided in a secondary health
of the following statements reflects a role of environment?
Clinical Nurse Specialist? A. Performing suctioning for a patient
A. Provides care and services under on a mechanical ventilator.
the supervision of an B. Performing the first wound dressing
anesthesiologist. for a patient in the unit after surgery.
B. Provides independent care, C. Conducting blood pressure
including pregnancy and screening for pregnant women at the
gynecological services Health Center.
C. Expert clinician in a specialized D. Teaching a patient diagnosed with
area of practice such as adult asthma on how to perform deep
diabetic care. breathing exercises
D. Provides comprehensive care,
usually in a primary care setting, 54. While doing your requirements as a
directly managing the medical care student nurse in the nurse's station, the staff
of patients who are healthy or have nurse on your duty suddenly had to leave
chronic conditions. the station immediately as one of her
patients was having code. You observed
51. Nurse Hiromi is presenting that the computer monitor displayed a
information to a management class of medical history of a patient not assigned to
nursing student regarding financial your care. What would be your most
reimbursement for achievement of appropriate action?
measurable patient outcomes. As a student A. Call the attention of your clinical
instructor and report the incidence.
B. Print the documents to serve as a A. These allow the staff nurses to
future reference, if in case the develop standardized integrated care plans
patient will be assigned to your care. for projected length of stay for patients of a
C. Turn off the computer as soon as specific case type.-
possible B. The exact contents and format of
D. Read the medical history for these must be the same among different
educational purposes. institutions to be standardized.
C. These delve in cases occur in high
55. What does HIPAA mandates to all volumes and are predictable
health care personnel with regards to D. These replace other nursing forms,
patient's records? A. Privacy such as the nursing care plans.- E.charting
B. Availability by exception is usually the method used.
C. Accessibility
D. Confidentiality 58. Nurse Kenji had been assigned to
patient Luke who had been admitted to the
56. Trisha, an irate patient admitted last hospital with chief complaints of gastric
week, informs the clerk, "This is too much! I upset and fever. The physician ordered
have paid a ton of money every time I come HNBB 20mg/mL IV q4h. Due to
to this hospital, even the clinics for routine incompetence, Nurse Kenji missed to verify
check-ups. Those medical records belong the doctor's order and administered HNBB
to me. I need them now!" What would be per Orem. He immediately reported this to
the best response to Trisha? his nursing supervisor and required him to
A. "Your original health care record put this in documentation. Which of the
belongs to the hospital. However, I following should not be considered when
may only give you the duplicate writing an incident report?
copies.' A. Assessment and judgment of the
B. "I am required to give you a request incident from your point of view.
form. This is intended to prove that B. Do not admit liability or give
you personally need your medical unnecessary details.
records, and not just anyone else." C. When writing the incident to the
C. "One moment please. Let me get patient's chart, do not mention the incident
them for you. Just relax and calm report. D. List the date, time, and care
yourself." given to patient and the name of the doctor
D. "I am so sorry, but you do not have to be notified.
the right to look at your own medical
records.' 59. Sashi, a new graduate nurse, is
employed in a public hospital utilizing the
57. Critical pathways are Focus Charting in nurse's notes. Which of
interprofessional care plans that identify the following statements regarding this
patient problems, key interventions, and format of documentation is/are correct: I.
expected outcomes within an established Some agencies include patient teachings-
time frame. As a student nurse, you know II. Data, action, response, patient teachings-
that these pathways are, except:
Ill. Response is the same as evaluation.- D. The nurse will administer pain
IV. Action combines planning and medication every 4 hours to keep the
implementation.- patient free from discomfort.
V. Must use all the steps each time you
make 63. A patient's son decides to stay at the
nurse's notes on particular focus. bedside while his father is confused. When
VI. Data is both subjective and objective developing the plan of care for this patient,
A. I, I, IV. V. VI what should the nurse do?
B. II, III, IV. V, VI A Individualize the care plan only according
C. I. II. III. V. VI — FEEL NAKO KANI. to the patient's needs
E CHECK LNG PLS. SA FDAR NI B. Request that the son leave at
D. I. II. Ill IV. VI bedtime, so the patient can rest.
C. Suggest that a female member of
60. Which of the following is considered the family stay with the patient.
as a Passive Strategy of health D. Involve the son in the plan of care as
promotion? A Smoking Cessation much as possible
Program
B. Fluoridation of Municipal Drinking 64. Risk factors play a major role in how a
Water nurse identifies a patient's health status.
C. Fitness Activity (Zumba Dance) every Which of the following is not considered as
5:00 PM a risk factor? A. Genetic predisposition
D. Weight Reduction Program B. Age
C. Poor diet
61. Nurse Jacob is presenting to the D Low education attainment
senior high school student covering the
facts about HIV awareness in a certain 65. In caring for a client with chronic
public school. disease, Nurse Star is performing skillfully
Which level of prevention is he practicing? the needed nursing interventions to alleviate
A. Primary Prevention the client's pain while preserving his dignity.
B. Secondary Prevention According to Swanson's Theory of Caring,
C. Tertiary Prevention this is included in what caring process?
D. Quaternary Prevention A. Knowing
B. Being With
62. A charge nurse is reviewing C. Doing For
outcome statements using the SMART D. Enabling
approach. Which patient outcome E. Maintaining Belief
statement will the charge nurse praise to
the new nurse? A. The patient will 66. Of the five caring processes described
ambulate in hallways. by Swanson, which describes "Being With"?
B. The nurse will monitor the patient's A. Providing for another as he would do for
heart rhythm continuously this shift. themselves
C. The patient will feed self at all B. Being emotionally there for another
mealtimes today without reports of person
shortness of breath.
C. Striving to understand an event as B. Being with the patient.
meaning for another person C. Being for the patient.
D. Sustaining faith in one's capacity to D. Being together with patient.
get through a situation
71. This is a form of nonverbal
67. A student nurse wants to clarify touch communication which influences a patient's
from a caring perspective. As a nurse, what security and comfort, enhances self-esteem,
will be your best response? increases confidence of health team &
A. "I do not touch the patient unless I am improves well-being.
performing a procedure or doing an A. Contact Touch
assessment. B. Noncontact Touch
B "Touch is only used when a patient is in C.. Task-Oriented Touch
pain." D. Protective Touch
C."Touch is a type of verbal communication e. Caring Touch
D. "Touch forms a connection between the
nurse and patient.
72. All of the following are guiding principles
68. Nurse Edward enters patient Bella's for the role of the nurse in the future of
room arranges the supplies for endotracheal patient care delivery, except?
suctioning, and explains the procedure to A. Managing the journey
her He informs the patient what to expect B. Relationships of virtual care presence a
during the procedure. Nurse Edward Knowledge is analyzed
proceeds to skillfully perform endotracheal C. The core of nursing is knowledge and
suctioning. This is an example of what type caring
of touch. D. None of the above
A.Protective Touch
B Task-Oriented Touch 73. All of the following serve as
C. Interpersonal Touch guidelines for quality documentation,
D. Caring Touch except? A Writing in detailed, narrative
format. B. Avoiding the use of jargons or
69. Which of the following is not an abbreviations.
outcome of nurses providing presence to C. Sticking to the facts.
patients? D. Use short sentences.
A. Alleviating suffering
B. Personal growth 74. Nurse Sky has been assigned as a
C. Connectedness reliever today. At 2:00 AM, a patient has
D. Decreasing sense of isolation and been rushed to the ER department with a
vulnerability case of Motor Vehicular Accident (MVA).
After gathering the patient's necessary data,
70. Nurse Rhoda is attending to Patient the nurse informed the doctor via telephone
Pinas. She is not only physically present, call. The doctor then provided numerous
but also understands the patient's orders to be done to the patient As a nurse,
concerns and health issues. She is which of the following guidelines must be
manifesting: A. Being there to the patient. considered in accepting telephone orders?
A. Read back any prescribed orders made B. "Patient arrived to ER via wheelchair
to the doctor. ??? with chief complaints of chest pain. Upon
B. Document every detail of the order into assessment, patient is restless and
the physician's order sheet. diaphoretic. VS taken and recorded on
C. Follow agency's protocols regarding PR sheet's
telephone orders C. "Patient arrived to ER at 12:03AM
D. Sign the complete name of the physician. due to chest pain"
D "Patient reported midsternal chest pain
75. In the problem-oriented medical record radiating to left shoulder starting 30mins
(POMR), which of the following is ago, pain scale of 8/10. Informed the AP
questionable in listing the problem? with patient's status and carried out orders
A. Team members list the problems in made. VS of BP: 180/100, HR: 102, RR:
alphabetical order. 22, T: 37.5°C and 02Sat of 97%. Endorsed
B. Team members add and date new to private room for further monitoring.'
problems as they arise.
C. The problem list includes a patient's 78. In developing communication skills,
physiological, psychological, social, Nurse Marco recognizes different opinions
cultural, spiritual, developmental, and that may conflict with his patient. This is
environmental needs. showing what aspect of attitude?
D. When a problem is resolved, the text A. Self-Confidence
of that problem is lined out B. Integrity
C. Humility
76. The nurse supervisor is assisting a new D. Critical Thinking
nurse on faxing a medical record of her E. Ethical Standards of Care
patient for immediate clinical needs. Which
of the following if done by the nurse must be 79. This is an assessment and
interrupted by her nurse supervisor? A. The communication technique that enables use
nurse uses a cover sheet. of self-awareness, motivation, empathy,
B. The nurse authenticates the data to social skills to build therapeutic relationship
verify the source and destination are with patients? A. Motivational Interview
correct after transmission. B. Narrative Approach
C. The nurse uses an encryption C. Emotional Intelligence
feature on the machine. D. Perceptual Biases
D. The nurse verifies the fax number of
the transmittal confirmation sheet 80. Student Nurse Xia is about to perform
NGTfeeding for the first time with her clinical
77. Which of the following documentation instructor's guidance. Before preparing the
entries is most accurate? equipment needed, she whispers "This is
A. "Patient arrived to ER clutching my opportunity to learn, and I can ask for
chest, restless, and diaphoretic. Reports help. This is what type of communication?
midsternal chest pain radiating to left A. Intrapersonal Communication
shoulder starting 30mins ago, pain scale of B. Interpersonal Communication
8/10. VS of BP: 180/100, HR: 102, RR: 22, C. Small-Group Communication
T: 37.5°C and 02Sat of 97%.' D. Public Community Communication
E. Electronic Communication of each tissue type (how it is organized)
and its function (what it does) B. There
81. All are considered negative is a relationship between tissue
feedback mechanism, except: A. Body structure and organ function.
temperature regulation C. Many diseases and pathologies are
B.Uterine contractions during delivery tissue and cell type specific.
C. Increase in blood pressure D.Changes at the tissue level affect the
D. Doing exercises. function of organs
E. All of these are correct.
82. Which of the following activities does F. All are correct except option C.
NOT represent an anatomical study:
A. making a section through the heart to 86. Which of these is NOT one of the
observe its interior four basic tissue types? A. glandular
B.examining the surface of a bone B. connective
C.viewing muscle tissue through a C. muscle
microscope D. epithelium
D. studying how the nerves conduct E. nervous
electrical impulses
e. observing the parts of a reproducing cell 87. Basement membranes:
A. Function as a filter and barrier to cell
83. Disruption in normal functioning of cells or pathogen movement
may result in death. One possible cause of B. Attach epithelial cells to the
disruption is the occurrence of disease. underlying tissue
A. The first statement is correct. The C. Are secreted partially by epithelial
second statement is wrong. cells and partially by the cells of the
B. The first statement is wrong. The second underlying tissue.
statement is correct. D. Function as a filter and barrier to cell
C. Both statements are correct. or pathogen movement, attach epithelial
D. Both statements are wrong. cells to the underlying tissue and are
secreted partially by epithelial cells and
84. Carrier mediated transport partially by the cells of the underlying tissue
mechanisms: are true of basement membranes. E. None
(Select all that apply) of these is correct.
A. Exhibit specificity.
B. Function to move small water-soluble 88. All are true statement about
molecules. epithelial tissue: (Select all that apply) A
C Function to move electrically charged Epithelial tissue is vascular B. Epithelial
ions. tissue provides physical protection for
D. Include facilitated diffusion, active organs.
transport, and secondary active transport. C. Epithelial tissue functions in
absorption & secretion.
85. Understanding tissue structure and D. Epithelial tissue controls the
function is important because A. There permeability into and out of an area/organ.
is a relationship between the structure
E. Epithelial tissue consists almost C. Axillary nerve
entirely of extracellular material between D.Radial nerve
them thereby forming a barrier against
entry into the body. 94. When a variation outside of normal
89. Which of these is not a factor that limits triggers an automatic response that
control stroke volume? corrects the situation, the mechanism is
A. sympathetic stimulation of the called:
myocardium A Homeostasis
B. end-diastolic volume B. positive feedback
C. end-systolic volume C. Crisis management
D. cardiac output D Negative feedback
E. contractility of the heart
95. The sciatic nerves arise from which
90. A client with pyrexia will most likely of these plexuses? A. Cervical
demonstrate: B. Choroid
A. Elevated blood pressure C. Lumbosacral
B. Dyspnea D. Brachial
C. Increased pulse rate
D. Precordial pain 96. Which part of the neuron
receives information? A. The axon
91. The Nurse explains to a client that B. The dendrite
the mechanism mediating long-term C. The Node of Ranvier
blood pressure regulation is the A. D. The myelin sheaths
Adjustment of urinary output
B. Fight or flight response 97. A role of immunoglobulin G (IgG) is to:
C. Capillary fluid shifts A. Activate the T-cell lymphocytes
D. Nervous system baroreceptors B. Activate the kinin system
C. Activate the complement system
92. The nurse is assisting a client to the D. Activate the process of inflammation -
bedside commode to urinate. Which brain MAO NI di man ma edit sa phonw
structures influence the client's bladder hahahaha
function? Select all that apply.
A. Brainstem, 98. Which of the following is an example of
B. Hypothalamus. naturally acquired active immunity?
C. Thalamus. A. A child receives the measles,
D. Cerebral cortex. mumps, and rubella (MMR) vaccine.
B. An adult contract a streptococcus
93. Compression of what nerve arising from infection and recovers in a couple of weeks.
the brachial plexus results in numbness, C. A child bitten by a rabid animal receives
tingling, and pain in the fingers, a condition an injection of rabies antibodies.
called carpal tunnel syndrome. A. Median D. Antibodies pass from mother to
nerve the developing child in the womb. E.
B. Musculocutaneous nerve All of the above.
99. The endocrine functions of the kidney
and the heart include the production and
secretion of the hormones:
A. insulin and glucagon
B. erythropoietin and atrial natriuretic peptic
C. epinephrine and norepinephrine
D. renin and angiotensinogen

100. Which of the following does not


stimulate the release of renin from the
kidneys?
A. a decline in renal blood flow
B. parasympathetic stimulation
C. sympathetic stimulation
D. none of the above
NP 2 A. Type of occupation: agricultural,
commercial, industrial
1. Which is the primary goal of community B. Location of the workplace in relation to
health nursing? health facilities
A. To support and supplement the C. Classification of the business enterprise
efforts of the medical profession in the based on net profit
promotion of health and prevention of D. Sex and age composition of employees
illness
B. To enhance the capacity of SITUATION: Field health services and
individuals families and communities to information system provides summary data
cope with their health needs on health service delivery and selected
C. To increase the productivity of the program from the barangay level up to the
people by providing them with services that national level. As a nurse, you should
will increase their level of health know the process on how this information
D. To contribute to national became processed and consolidated.
development through promotion of family
welfare, focusing particularly on mothers 5. All of the following are objectives of
and children. FHSIS Except?
A. To complete the clinical picture of
2. CHN is a community-based practice. chronic disease and describe their natural
Which best explains this statement? history
A. The service is provided in the natural B.To provide standardized, facility level
environment of people. data accurate and are disseminated in a
B. The nurse has to conduct community timely and easy to use fashion
diagnosis to determine nursing needs and C. To minimize recording and reporting
problems. burden allowing more time for patient care
C. The services are based on the and promotive activities
available resources within the community. D. To ensure that data reported are
D. Priority setting is based on the useful and accurate and are disseminated
magnitude of the health problems in timely and easy to use fashion
identified.
6. What is the fundamental block or
3. Population-focused nursing practice foundation of the field health service
requires which of the following processes? information system?
A. Community organizing A. Family treatment record
B. Nursing process B. Target Client list
C. Community diagnosis C.Reporting forms
D. Epidemiologic process D.Output record

4. What other factor must be 7. What is the primary advantage of having


considered in determining the occupational a target client list?
health privileges to which the workers will A. Nurses need not to go back to FTR to
be entitled? monitor treatment and services to
beneficiaries thus saving time and effort B.
Help monitor service rendered to clients in 12. As a newly appointed PHN instructed to
general organize Barangay Baritan, which of the
C. Facilitate monitoring and supervision of following is your initial step in organizing the
services community for initial action?
D. Facilitate easier reporting A. Study the Barangay Health statistics and
records
8. Which of the following is used to monitor B. Make a courtesy call to the Barangay
particular groups that are qualified as Captain
eligible to a certain program of the DOH? C. Meet with the Barangay Captain to make
A. Family treatment record plans
B. Target Client list D. Make a courtesy call to the Municipal
C. Reporting forms Mayor
D. Output record
13. Preparatory phase is the first phase in
9. In using the tally sheet, what is the organizing the community. Which of the
recommended frequency in tallying activities following is the initial step in the preparatory
and services? A. Daily phase? A. Area selection
B. Weekly B. Community profiling
C.Monthly C. Entry in the community
D. Quarterly D. Integration with the people

10. After bringing the reportin unig forms in 14. The most important factor in
the right facility for processing, Nurse Budek determining the proper area for community
knew that the output reports are solely organizing is that this area should
produced by what office? A Rural health A. Be already adopted by another
office organization
B. FHSIS Main office B. Be able to finance the projects
C. Provincial health office C. Have problems and needs assistance D.
D. Regional health office Have people with expertise to be
developed as leaders
SITUATION: Community organizing is a
process by which people, health services 15. Which of the following dwelling place
and agencies of the community are brought should the Nurse choose when integrating
together to act and solve their own with the people?
problems. A. A simple house in the border of Barangay
Baritan and San Pablo
11. Mang ambo approaches you for B. A simple house with fencing and gate
counseling. You are an effective counselor if located in the center of Barangay Baritan
you C. A modest dwelling place where people
A. Give good advice to Mang Ambo will not hesitate to enter
B. Identify Mang Ambo's problems C. D. A modest dwelling place where people
Convince Mang Ambo to follow your will not hesitate to enter located in the
advice center of the community
D. Help Mang Ambo identify his problems
16. In choosing a leader in the A. Make a lesson plan
community during the Organizational B. Set learning goals and objective
phase, which among these people will C. Assess their learning needs
you choose? A. Miguel Zobel, 50 years D. Review materials needed for training
old, Rich and
Famous 21. Nurse Budek is conducting a health
B. Rustom, 27 years old, Actor C. Mang teaching to Agnesia, 50-year-old breast
Ambo, 70, Willing to work for the desired cancer survivor needing rehabilitative
change measures. He knows that health education
D. Ricky, 30 years old, Influential and is effective when
Willing to work for the desired change A. Agnesia recites the procedure and
instructions perfectly
17. Which type of leadership style should B. Agnesia's behavior and outlook in
the leaders of the community practice? life was changed positively
A. Autocratic C. Agnesia gave feedback to Budek
B. Democratic saying that she understood the instruction
C. Laissez Faire D. Agnesia requested a written instruction
D. Consultative from Budek

18. Setting up Committee on Education and 22. Which of the following is true about
Training is in what phase of COPAR? health education?
A. Preparatory A. It helps people attain their health through
B. Organizational the nurse's sole efforts
C. Education and Training B. It should not be flexible
D. Intersectoral Collaboration C. It is a fast and mushroom like process D.
E. Phase out It is a slow and continuous process.

19. Community diagnosis is done to come 23. Which of the following factors least
up with a profile of local health situation that influence the learning readiness of an
will serve as basis of health programs and adult learner? A. The individual's stage of
services. This is done in what phase of development B. Ability to concentrate on
COPAR? information to be learned
A. Preparatory C. The individual's psychosocial
B. Organizational adaptation to his illness
C. Education and Training D. The internal impulses that drive the
D. Intersectoral Collaboration person to take action
E. Phase out
24. Which of the following is the most
20. The people named the community important condition for diabetic patients to
health workers based on the collective learn how to control their diet?
decision in accordance with the set criteria. A. Use of pamphlets and other
Before they can be trained by the Nurse, materials during instructions B.
The Nurse must first Motivation to be symptom free C.
Ability of the patient to understand
teaching instruction D. Language used 29. If Regielyn will compute for the monthly
by the nurse doses of OPV, she will have which of
the following:
25. An important skill that a primigravida A. 115
has to acquire is the ability to bathe her B. 198 C. 151
newborn baby and clean her breast if she D. 132
decides to breastfeed her baby, which of the
following learning domain will you classify 30. Compute for the monthly doses'
the above goals? requirement for Hepatitis B vaccine:
A. Psychomotor A. 130 B. 151
B. Cognitive C. 132
C. Affective D. 115
D. Attitudinal
Situation: According to study,
Situation: Community health nurse Regielyn environmental health as well as
will administer immunization to children in environmental sanitation is still a concern
her assigned community. The total and a problem in our country.
population of Barangay Sta. Mesa is 15,750
people. The right distribution depends on 31. The Department of Health through
the availability of vaccines and the Environmental and Occupational Health
appropriate health personnel. Office has authority to act on all issues
related to environment and health. Nurse
26. Regielyn is required to have her Francis is aware that diarrheal diseases
monthly doses for Tetanus Toxoid (TT). ranked first as the leading cause of disease
She will come up with which of the following among general population. The
choices: comprehensive sanitation code of the
A 79 Philippines is mandated and put into law
B. 198 as:
C. 99 A. PD 825
D. 153 B. RA 8749 C. PD 856
D. RA 9275
27. What will be the eligible population for
Hepa, B vaccine? 32. Neil, a newly hired public health worker
A. 551 B. 550 C. 472 is no working in his assigned
D. 473 community, Barangay Bidin, in a remote
area under the Municipality of Sibutu.
28. Compute for the monthly doses of Anti- During his community assembly, he
Measles Vaccine: emphasized that their different types of
A. 79 toilets that are being utilized in the said
B. 130 barangay. The EHS set policies on
C. 99 approved types of toilets and
D. 68 categorizing them according to different
levels. Neil has correct understanding
regarding toilet level - Il if he states that.
A. "Level I is a non-water carriage toilet 34. There are 4 rights in food safety. These
facility; no water is necessary to wash rights involve the chain in food processing
the waste into the receiving space this from the source in the market until the food
include pit latrines reaches the table. Neil, knows that all are 4
B "Level Il is a type of toilet facility requiring rights in food safety except:
small amount of water to wash the waste
into the receiving space like pour flush' A Right Source
C. "Level Il is a water carriage type of toilet B. Right Cooking
facility that is connected to a septic tank C. Right Food
and to a sewerage system to treatment D. Right Storage
plant"
D "Level I is an on-site toilet facility of a 35. The modern-day practice of water
water carriage type with water-sealed and supply among urban areas is through water
flush type with septic tank" refilling station. Andy Lynn, as an owner of a
water refilling distilled station knows that
33. The Department of Health (DOH) she has to regularly monitor the quality of
through the EOHO has set some policies on drinking water being sold in her station, if
the following areas such as (1) approved she will submit her water for analysis for
types of water facilities (2) Unapproved physical and chemical properties, it should
types of water facilities (3) Access to safe be submitted every how many months:
and potable drinking water (4) water quality
and monitoring surveillance and (5) A. Every month
waterworks system and well construction. B. Every 6 months
Leila, knows that Level I approved type of C. Annually
water supply facility include all of the D. As need arises
following choices apart from one that is
incorrect: 36. Which of the following statements are
A. Designed to deliver 40 - 80 liters of true?
water, to an average of 100 households, A. All communicable diseases are
with one faucet per 4-6 households B. infectious and contagious.
Suitable for rural areas where houses are B. All contagious diseases are
clustered densely to justify a simple piped infectious but not all infectious are
system.- contagious. C. Communicable diseases are
C. Communal Faucet System or Stand - all contagious but not infectious.
Posts, a system composed of a source, a D.Infectious diseases are contagious and
reservoir, a piped distribution network and also communicable.
a communal faucet, located not more than 37. An organism that is capable of invading
25 meters from the farthest house D. A and multiplying in the body of the host:
protected well or a developed spring with A. Causative Agent
an outlet but without distribution system, B. Reservoir
generally adaptable for rural areas where C. Bacteria
the house is thinly scattered. D. Carrier
38. The infecting ability of a C. Given at a dose of 10 mg/kg twice a
microorganism depends on its degree of: day for two days, then 5mg/kg single dose
A. Pathogenicity on the third day
B. Communicability D. Given at a dose of 5 mg/kg twice a
C. Teratogenicity day for two days, then 5mg/kg single dose
D. Epidemiology on the third day

39. Amazingly, patients with malaria have a 43. Which of the following health teachings
cycle of cold, hot and diaphoretic stage. will be included if the patient is taking
Fever is evident in a particular period of chloroquine?
time and not all through the disease process A. Advise patient that the urine may turn
because: rust brown in color. B. It should be given
A. During febrile stage toxins are before meals.
present.* B. During febrile stage, the C. Can be giiven even before c and S is
microorganisms simultaneously completed or confirmed.
reproduce . D. Frequent ambulation is required
C. During febrile stage, iron is when taking the medication.
sequestered by the liver.
D. During febrile stage, the 44. Which of the following manifestations of
gametocytes are produced. the client, that according to WHO, may lead
to definitive diagnosis of Dengue
40. A diagnostic procedure that is a must Hemorrhagic fever, except:
in every case of Malaria in a certain A. Fever lasting 2-7 days
community: A. Malarial Blood Smear B. Thrombocytopenia
B. Quantitative Buffy Coat C. Positive tourniquet test plus petechiae,
C. ELISA ecchymoses, purpura
D. Nocturnal Blood Examination D.Abdominal pain plus joint fever

41. The drug of choice against malaria is: 45. In what stage is Herman's sign
A. Chloroquine clinically manifested by the client? A
B. N-diethyl-D-toluamide Febrile stage
C. Pyethrinoids B.Hemorrhagic Stage
D. Biltricide C. Circulatory failure stage
D. Hypovolemic Shock
42. Blood Schizonticides are given with the
following dose: 46. A child is admitted to the pediatric unit
A. Given at a dose of 10 mg/kg once a with a diagnosis of suspected
day for two days, then 5rg/kg single dose meningococcal meningitis. Which of the
on the third day. following nursing measures should the
B. Given at a dose of 5 mg/kg once a nurse do FIRST?
day for two days, then 10 mg/kg single A. Institute seizure precautions
dose on the third day B.Assess neurologic status
C. Place in respiratory isolation
D. Assess vital signs
47. A client is diagnosed with methicillin 51. A nurse is describing the process of
resistant staphylococcus aureus fetal circulation to a client during a prenatal
pneumonia. What type of isolation is visit. The nurse accurately tells the client
MOST appropriate for this client? A. that fetal circulation consists of:
Reverse isolation A. Two umbilical veins and one
B. Respiratory isolation umbilical artery
C. Standard precautions B. Two umbilical arteries and one
D. Contact isolation umbilical vein
C. Arteries carrying oxygenated blood
48. Several clients are admitted to an adult to the fetus
medical unit. The nurse would ensure D. Veins carrying deoxygenated blood
airborne precautions for a client with which to the fetus
of the following medical conditions? A. A
diagnosis of AIDS and cytomegalovirus B. 52. A nurse prepares to assess a fetal
A positive PPD with an abnormal chest x- heartbeat. The nurse uses a fetoscope,
ray knowing that the fetal heartbeat can first be
C. A tentative diagnosis of viral pneumonia heard with a fetoscope at gestational week:
D. Advanced carcinoma of the lung A. 5
B. 10 C. 16
49. Which of the following is the FIRST D. 20
priority in preventing infections when
providing care for a 53. A nurse is performing an
A. Hand washing assessment of a primipara who is being
B. Wearing gloves evaluated in a clinic during her second
C. Using a barrier between client's furniture trimester of pregnancy. Which of the
and nurse's bag D. Wearing gowns and following indicates an abnormal physical
goggles finding necessitating further testing?
A. Consistent increase in fundal height
50. An adult woman is admitted to an B. Fetal heart rate of 180 beats per
isolation unit in the hospital after minute
tuberculosis was detected during a pre- C. Braxton hicks' contractions
employment physical. Although frightened D. Quickening
about her diagnosis, she is anxious to 54. A nurse is performing an
cooperate with the therapeutic regimen. assessment of a pregnant client who is at
The teaching plan includes information 28 weeks of gestation. The nurse
regarding the most common means of measures the fundal height in centimeters
transmitting the tubercle bacillus from one and expects the findings to be which of the
individual to another. Which contamination following?
is usually responsible? A.22 cm
A. Hands B. B.28 cm
Droplet nuclei. C.36 cm
C.Milk products. D.40 cm
D.Eating utensils
55. A pregnant client asks the nurse in encouraged to walk. When she returns to
the clinic when she will be able to start bed for a monitor check, she complains of
feeling the fetus move. The nurse responds an urge to push. When performing a vaginal
by telling the mother that fetal movements examination, the nurse accidentally ruptures
will b noted between: A. 6 and 8 weeks of the amniotic membranes, and as she
gestation withdraws her hand, the umbilical cord
B.8 and 10 weeks of gestation comes out. What should the nurse do next?
C.10 and 12 weeks of gestation A. Put the client in a knee-to-chest position
D. 14 and 16 weeks of gestation B. Call the physician or midwife
C. Push down on the uterine fundus D.
56. A clinic has instructed a pregnant client Set up for a fetal blood sampling to
in measures to prevent varicose veins assess for fetal acidosis
during pregnancy. Which statement if made
by the client indicates a need for further 60. The nurse can consider the fetus's head
education? to be engaged when:
A. "I should wear support hose" B. "I A. The presenting part moves through
should be wearing flat nonslip shoes that the pelvis
have an arch support* B. The fetal head rotates to pass
C. "I should wear pantyhose" through the ischial spines
D. "I can wear knee-high hose as long as I C. The fetal head extends as it passes
don't leave them on longer than 8 hours. under the symphysis pubis
D. The biparietal diameter passes the
57. The nurse is providing care for a pelvic inlet
postpartum client. Which of the following
condition would place this client at greater 61. A client is experiencing a true labor
risk for a postpartum hemorrhage? when her contraction pattern shows
A Hypertension A. Occasional irregular contractions B.
B. Uterine infection Irregular contractions that increase in
C. Placenta previa intensity
D Severe pain C. Regular contractions that remain the
same
58. During the 3'd postpartum day, which of D. Regular contractions that increase in
the following would the nurse be most likely frequency and duration
to find in the client? 62. When late decelerations are noted
A. She's interested in learning more by the nurse, the first action is to: A.
about newborn care Notify the physician STAT
B. She talk a lot about her birth B. Position the client on her left side
experience C. She sleeps whenever the C. Administer oxygen via face mask D.
baby isn’t present Increase the drip rate of the intravenous
D. She requests help in choosing a name fluid
for the baby
63. A client is receiving magnèsium sulfate
59. A client in the active phase of labor has to help suppress preterm labor. The nurse
a reactive fetal monitor strip and has been
should watch for which sign of magnesium A. Closed anterior fontanel and open
toxicity. posterior fontanel.----
A. Headache B. Open anterior fontanel and closed
B. Loss of deep tendon reflexes posterior fontanel.--- dapat Kani mn ata
C. Palpitations C. Closed anterior and posterior
D. Dyspepsia fontanels. Kani sure? Oo kani pud D.
Open anterior and posterior fontanels.
64. During prenatal screening of a diabetic
client, the nurse should keep in mind that 68. An infant is diagnosed with patent
the client is at increased risk for: ductus arteriosus. Which of the following
A. Rh incompatibility drugs may be administered in hopes of
B. Placenta previa achieving pharmacologic closure of the
C. Hyperemesis defect? A. Digoxin (Lanoxin)
D. Stillbirth B. Prednisone
C. Furosemide (Lasix)
65. Which of the following is a D. Indomethacin (Indocin)
normal physiological response in the
early postpartum period? A. Urinary 69. The nurse is teaching a mother about
urgency the benefits of breast-feeding her infant.
B. Rapid diuresis Which type of immunity is passed on to the
C. Decrease in blood pressure infant during breast-feeding? A Natural
D.Increased motility of the Gl system immunity
B Natural acquired active immunity
66. A newly pregnant client who is a little C. Naturally acquired passive immunity
overweight ask how much weight she D. Artificially acquired active immunity
should gain over the 9 months. The most
appropriate answer is; 70. The nurse would anticipate a possible
A. "For your size a little heavy, about complication in infants delivered by
15 pounds would be best cesarean section. This condition would be:
B. "It really doesn't matter exactly how A. Respiratory distress
much weight you gain, as long as your diet B Renal impairment
is healthy. C AB incompatibility
C. "A gain of about 24-25 pounds is D. Kernicterus
best for mother and baby. Sure ni? Oo na
search ni nako 71. An 11 lb. 6 oz. baby girl was delivered
D. "Because you are a little overweight, by cesarean section to a diabetic mother.
it would be best for you not to gain too The priority assessment of the infant of a
much weight.---- diabetic mother would be for;
A. Hypoglycemia
67. A parent brings a 19-month-old toddler B. Sepsis
to the clinic for a well-child checkup. When C. Hyperglycemia
palpating the toddler's fontanels, the nurse D. Hypercalcemia
would expect to find:
72. During a physical exam of an infant with immediately wants to be held. What is the
congenital hip dysplasia, the nurse would best interpretation of this behavior?
observe and report which of the following A. The toddler isn't effectively coping
characteristics? with stress.
A. Symmetrical gluteal folds B. The toddler's need for affection isn't
B. Limited adduction of the affected leg C. being met.
Femoral pulse when the hip is flexed and C. This is normal behavior for a 2-year-
the leg is abducted old child.
D. Limited abduction of the affected leg D. This behavior suggests the need for
counseling.
73. The nurses caring for a premature baby
use careful hand washing techniques 76. Which nursing intervention has priority
because they know premature infants are when feeding an infant with a cleft lip or
more susceptible to infection than full-term palate?
infants. Which of the following explains why A. Directing the flow of milk in the center of
premature infants are more likely to develop mouth
infection? B. Providing small, frequent feedings
A. Their liver enzymes are immature B. C. Avoiding breast-feeding
Premature babies may receive steroid D. Infrequent burping
drugs, which affects the immune system
C. Premature infants receive few 77. Which action best explains the main
antibodies from the mother, because role of surfactant in the neonate?
antibodies pass across the placenta A. Assist with ciliary body maturation in
during the last month of pregnancy the upper airways
D. Surfactant is decreased in premature B. Helps maintain a rhythmic breathing
infants pattern
C. Promotes clearing mucus from the
74. In the delivery room, a client has just D. Helps the lungs remain expanded atter
delivered a healthy 7-pound baby boy. The the initiation of breathing
physician instructs the nurse to suction the
baby. The procedure that the nurse would 78. While assessing a 2-hour-old neonate,
use is to: the nurse observes the neonate to have
A. Suction the nose first acrocyanosis. Which of the following
B Suction the mouth first nursing actions should be performed
C. Suction neither the nose nor mouth initially?
until the physician gives further A. Activate the blue code or
instructions D. Turn the baby on his side emergency system
so mucus will drain out before suctioning B. Do nothing because acrocyanosis
is normal in the neonate C. Immediately
75. A mother tells the nurse that her 22- take the neonate's temperature according
month-old child says "no" to everything. to hospital policy D. Notify the physician of
When scolded, the toddler becomes angry th e need for a cardiac consult
and starts crying loudly, but then
79. When performing a neurologic C Administer 0.3ml/kg of epinephrine
assessment, which sign is considered a D. Do nothing
normal finding in a neonate?
A. Doll eyes 84. Resuscitation in a newborn may cause
B. "Sunset eyes many side effects including hypovolemia.
C. Positive Babinski's sign Which of the following is not included of the
D. Pupils that don't react to light signs and symptoms? A. Hypotension
B. Bradycardia
80. A client's mother asks the nurse why C. Decrease CVP
her newborn grandson is getting an D. Decrease peripheral tissue perfusion*
injection of vitamin K. Which best explains
why this drug is given to neonates? A. 85. A nurse in a delivery room is assisting
Vitamin K assists with coagulation with the delivery of a newborn infant. After
B. Vitamin K assists the gut to mature the delivery, the nurse prepares to prevent
C. Vitamin K initiates the immunization heat loss in the newborn resulting from
process evaporation by:
D. Vitamin K protects the brain from excess A Warming the crib pad
fluid production B. Turning on the overhead radiant warmer
C Closing the doors to the room
81. Respiratory depression in neonates that D. Drying the infant in a warm blanket
is due to maternal use of narcotics is given
a narcotic antagonist which is Naloxone 86. In regulating the temperature of the
(Narcan) through IM. What is the other route newborn which of the following action of the
for administering Narcan? nurse requires further attention? A. Wiping
A ID the newborn dry B. Placing newborn under
B. Tropical a prewarmed radiant warmer
C. Oral C. Initiating skin-skin contact
D. Umbilical Vessel D. Placing newborn on X-ray tables
87. Radiant heat warmers are open beds
82. During assessment of a newborn, what with overhead radiant source used in
is the 1st sign of obstruction or respiratory regulating the newborns' temperature. It
compromise? contains a probe which is tape in what site?
A Increasing RR A Head
B. Increasing PR B. Xiphoid Process?*
C. Increasing BUN C. Upper extremities
D. Increasing Creatinine D. Lower extremities

83. A newborn that has no audible 88. Asphyxia is a condition arising when
heartbeat is being resuscitated in the the body is deprived of oxygen, infants
delivery room. After 30 seconds of CPR the who have this condition usually receive IF
heart rate is 70 bpm. The next action of the to prevent exhaustion. Furthermore, an
nurse would be to? A. Administer 0.5 ml/kg infant who also is tachypneic receives
of epinephrine gavage feeding which also needs to be
B. Administer 1ml/kg of epinephrine given a pacifier at feeding times. Which of
the following is a contraindication in giving 93. Baby girl Gretch was born large for
a pacifier? gestational age. After being delivered
A NEC vaginally, this infant should be carefully
B. Good sucking reflex assessed for:
C. Intussusception A. Increased intracranial pressure
D. TEF B. Hypothermia
C. Decreased red blood levels (Anemia)
89. A pregnant woman's blood test D. Hyperglycemia
revealed a very low hemoglobin level. The
physician considers the woman severely 94. There are many conditions associated
anemic. Which of the following is the most with macrosomic infants, which of the
likely effect on the fetus if the woman is following refers to an overgrowth disorder?
severely anemic during pregnancy? A. A. PDA
Hemorrhage could be possiblel B. Omphalocele
B. Erythroblastosis fetalis C. Beckwith-Wiedemann Syndrome
C. Small for Gestational Age (SGA) baby D. DM
D. Large for Gestational Age (LGA) fetus
95. What is the preferred birthing type for
90. Small for Gestational Age infants fall LGA infants?
below 10TH percentile of the normal weight. A Normal Spontaneous Vaginal Delivery
Which of the following will you assess in B. Forceps Delivery
these infants? A Good skin turgor C Assisted Delivery
B Normal Liver D. Cesarean Section
C. Large head 96. A cardiovascular dysfunction that
D. Intact skull sutures happens to GA infants that involves a group
91. Why are small for gestational age of congenital heart defects involving an
newborns are at risk for difficulty abnormal spatial arrangement of any of the
maintaining body temperature? great vessels. A. PDA**
A. They are preterm so are born B. Tetralogy of fallot
relatively in small size C. Transposition of great vessels*
B. They are more active than usual so D. CHF
throw off covers
C. They do not have as many fat store 97. Neonates of mothers with diabetes are
as other infants at risk for which complication following
D. Their skin is more susceptible to birth?
conduction of cold A. Atelectasis
B. Microcephaly
92. This is the blueness of extremities C. Pneumothorax
observed in Small for Gestational Infants. D. Macrosomia
A. Acrocyanosis B.
Cyanosis 98. In infants with DM mothers, what
C. Hemosiderosis happens to their insulin needs at birth?
D. Kernicterus A Increase
B. Decrease D.Thrombosis
C Normalize 100. A preterm infant is those born
D. Fluctuates before the end of week 37 of gestation
and what another criterion? A. Weight
99. The nurse is aware that a neonate of >2,500 g (5 1b 8oz)
a mother with diabetes is at risk for what B. Weight < 2,500 g (5 Ib 8 oz)
complication? A. Anemia C. Weight < 2,500 g (8 lb 5 oz)
B. Hypoglycemia D.Weight > 2,500 g (8lb 5oz)
C. Nitrogen loss
NP 3 doctor orders an ECG. The result below will
be interpreted
1.The nurse suspects that her client is in as
cardiac arrest. According to the American
Heart Association, the nurse should perform
the actions listed below. Order these actions
in the sequence that the nurse should
perform them.
A. AV bundle block
I. Activate the emergency medical system. B. Sinus rhythm
II. Assess responsiveness. C. Atrial flutter
III. Call for a defibrillator. D. Sinus arrhythmia
IV. Provide two slow breaths. V.
Assess pulse. 4. The electrocardiogram (ECG) tracing
VI. Assess breathing. shown below, excluding the seventh beat,
has a normal QRS complex, one premature
A. 213645 atrial contraction (PAC), and what other
B. 132456 attributes?
C. 1,2,3,6,5,4
D. 2,3.1,5,6,4

2. A client comes to the ER complaining of


chest pain. An electrocardiogram (ECG)
reveals myocardial ischemia and an anterior A. P wave is identifiable, PR is 0.16 second,
wall myocardial infarction (MI). Which ECG and sinus rhythm is at 95 beats/minute.
characteristics indicates myocardial B. P wave and PR interval are unidentifiable
ischemia? and sinus arrhythmia is at 95
A. Prolonged PR interval beats/minute.
B. Absent Q wave C. P wave is identifiable, PR interval is 0.16
C. Elevated ST Segment second, and sinus rhythm is at 95
D.Widened QRS complex beats/minute.
D. P wave is identifiable, PR interval is
3. Nyosh admits himself on the ER 0.16 second, and sinus arrhythmia is at
following an intense chest pain and the 95 beats/minute. //Way sure ni
5. A client who is awake and diaphoretic Excrete excessive fluids accumulated at
has a palpable pulse. The nurse runs an night
electrocardiogram (ECG) strip, which C. Prevents sleep disturbances during night
reveals the following pattern for the lead I. D. Prevention of electrolyte imbalance
What does the pattern indicate?
10. What would be the primary goal of
A. Normal sinus rhythm therapy for a client with pulmonary edema
B. Atrial fibrillation and heart failure?
C. Ventricular standstill A. Enhance comfort
D. Ventricular tachycardia B. Increase cardiac output
C. Improve respiratory status
6. Mrs. Chua a 78-year-old client is D. Peripheral edema decreased
admitted with the diagnosis of mild chronic
heart failure. The nurse expects to hear 11. Nurse Linda is caring for a client with
when listening to client's lungs indicative of head injury and monitoring the client with
chronic heart failure would be: decerebrate posturing. Which of the
following is a characteristic of this type of
A. Stridor posturing?
B. Crackles A. Upper extremity flexion with lower
C. Wheezes extremity flexion
D. Friction rubs B. Upper extremity flexion with lower
extremity extension
7. Patrick who is hospitalized following a C. Extension of the extremities after a
myocardial infarction asks the nurse why he stimulus
is taking morphine. The nurse explains that D. Flexion of the extremities after stimulus
morphine
A. Decrease anxiety and restlessness 7. A female client is taking Cascara
B. Prevents shock and relieves pain Sagrada. Nurse Betty informs the client that
C. Dilates coronary blood vessels the following maybe experienced as side
D. Helps prevent fibrillation of the heart effects of this medication: A. GI bleeding
B. Peptic ulcer disease
8. Which of the following should the nurse C. Abdominal cramps
teach the client about the signs of digitalis D. Partial bowel obstruction
toxicity?
A. Increased appetite 8. Dr. Marquez orders a continuous
B. Elevated blood pressure intravenous nitroglycerin infusion for the
C. Skin rash over the chest and back D. client suffering from myocardial infarction.
Visual disturbances such as seeing Which of the following is the most essential
yellow spots nursing action?
A. Monitoring urine output frequently
9. Nurse Trisha teaches a client with heart B. Monitoring blood pressure every 4 hours
failure to take oral Furosemide in the C. Obtaining serum potassium levels daily
morning. The reason for this is to help… D. Obtaining infusion pump for the
A. Retard rapid drug absorption B. medication
9. During the second day of hospitalization oxygen content of different heart
of the client after a Myocardial Infarction. chambers
Which of the following is an expected
outcome? 13. During the first several hours after
A. Able to perform self-care activities a cardiac catheterization, it would be
without pain most essential for nurse Cherry to…
B. Severe chest pain A. Elevate client's bed at 45° B.
C. Can recognize the risk factors of Instruct the client to cough and deep
Myocardial Infarction breathe every 2 hours
D. Can Participate in cardiac rehabilitation C. Frequently monitor client's apical pulse
walking program and blood pressure
10. A 68-year-old client is diagnosed with a D. Monitor client’s temperature every hour
right-sided brain attack and is admitted to
the hospital. In caring for this client, the 14. Kate who has undergone mitral valve
nurse should plan to: replacement suddenly experiences
A. Application of elastic stockings to prevent continuous bleeding from the surgical
flaccid by muscle incision during postoperative period. Which
B. Use hand roll and extend the left upper of the following pharmaceutical agents
extremity on a pillow to prevent should Nurse Aiza prepare to administer to
contractions Kate?
C. Use a bed cradle to prevent dorsiflexion A. Protamine Sulfate
if feet B. Quinidine Sulfate
D. Do passive range of motion exercise C. Vitamin C
D. Coumadin
11. Nurse Liza is assigned to care for a
client who has returned to the nursing unit 15. In reducing the risk of endocarditis,
after left nephrectomy. Nurse Liza’s good dental care is an important measure.
highest priority would be… A. Hourly To promote good dental care in client with
urine output mitral stenosis in teaching plan should
B. Temperature include proper use of… A. Dental floss
C. Able to turn side to side B. Electric toothbrush
D. Able to sips clear liquid C. Manual toothbrush
D. Irrigation device
12. A 64-year-old male client with a long
history of cardiovascular problem including 16. Among the following signs and
hypertension and angina is to be scheduled symptoms, which would most likely be
for cardiac catheterization. During pre present in a client with mitral regurgitation?
cardiac catheterization teaching, Nurse A. Altered level of consciousness
Cherry should inform the client that the B. Exceptional Dyspnea C. Increase
primary purpose of the procedure is… creatine phospholinase
A. To determine the existence of CHD concentration
B. To visualize the disease process in D. Chest pain
the coronary arteries C. To obtain the
heart chambers pressure D. To measure
17. Kris with a history of chronic infection of D. Erythrocyte count
the urinary system complains of urinary
frequency and burning sensation. To figure 22. A client is admitted with a diagnosis
out whether the current problem is in renal of polycythemia vera. The nurse should
origin, the nurse should assess whether the closely monitor the client for: A.
client has discomfort or pain in the… Increased blood pressure
A. Urinary meatus B. Decreased respirations
B. Pain in the Labium C. Increased urinary output
C. Suprapubic area D. Decreased oxygen saturation
D. Right or left costovertebral angle
18. Nurse Perry is evaluating the renal 23. The doctor has prescribed a diet high in
function of a male client. After documenting vitamin B12 for a client with pernicious
urine volume and characteristics, Nurse anemia. Which foods are highest in vitamin
Perry assesses which signs as the best B12?
indicator of renal function. A. Meat, eggs, dairy products
A. Blood pressure B. Peanut butter, raisins
B. Consciousness C. Broccoli, cauliflower, cabbage
C. Distension of the bladder D. Shrimp, legumes, bran cereals
D. Pulse rate
24. Which of the following arteries
19. John suddenly experiences a primarily feeds the anterior wall of the
seizure, and Nurse Gina notice that John heart? A. Circumflex artery
exhibits uncontrollable jerking B. Internal mammary artery
movements. Nurse Gina documents that C. Left anterior descending artery
John experienced which type of seizure? D. Right coronary artery
A. Tonic seizure
B. Absence seizure 25. When do coronary arteries
C. Myoclonic seizure primarily receive blood flow? A. During
D. Clonic seizure inspiration
B. During diastole
20. Smoking cessation is critical strategy for C. During expiration
the client with Burgher's disease, Nurse D. During systole
Jasmin anticipates that the male client will
go home with a prescription for which 26. Which of the following illnesses is
medication? A. Paracetamol B. Ibuprofen the leading cause of death in the US?
C. Nitroglycerin A. Cancer
D. Nicotine (Nicotrol) B. Coronary artery disease
C. Liver failure
21. Which of the following lab studies D. Renal failure
should be done periodically if the client is
taking warfarin sodium (Coumadin)? A. 27. Which of the following conditions most
Stool specimen for occult blood commonly results in CAD?
B. White blood cell count A. Atherosclerosis
C. Blood glucose
B. DM C. Oral medication administration D.
C. MI Percutaneous transluminal coronary
D. Renal failure angioplasty are invasive, surgical
treatments.
28. Atherosclerosis impedes coronary
blood flow by which of the following 33. Prolonged occlusion of the right
mechanisms? coronary artery produces an infarction in
A. Plaques obstruct the vein which of the following areas of the heart?
B. Plaques obstruct the artery A. Anterior
C. Blood clots form outside the vessel wall B. Apical
D. Hardened vessels dilate to allow the C. Inferior
blood D. Lateral
34. After an anterior wall myocardial
29. Which of the following risk factors for infarction, which of the following problems is
coronary artery disease cannot be indicated by auscultation of crackles in the
corrected? lungs?
A. Cigarette smoking A. Left-sided heart failure
B. DM B. Pulmonic valve malfunction C.
C. Heredity Right-sided heart failure
D. HPN D. Tricuspid valve malfunction

30. Exceeding which of the following serum 35. Which of the following diagnostic tools
cholesterol levels significantly increases the is most commonly used to determine the
risk of coronary artery disease? location of myocardial damage?
A. 100 mg/dl A. Cardiac catheterization
B. 150 mg/dl B. Cardiac enzymes
C. 175 mg/dl C. Echocardiogram
D. 200 mg/dl D. Electrocardiogram

31. Which of the following actions is the first 36. What is the first intervention for a client
priority care for a client exhibiting signs and experiencing myocardial infarction?
symptoms of coronary artery disease? A. A. Administer morphine
Decrease anxiety B. Administer oxygen
B. Enhance myocardial oxygenation C. Administer sublingual nitroglycerin
C. Administer sublingual nitroglycerin D. Obtain an electrocardiogram
D. Educate the client about his symptoms
37. What is the most appropriate nursing
32. Medical treatment of coronary response to a myocardial infarction client
artery disease includes which of the who is fearful of dying?
following procedures? A. Cardiac A. “Tell me about your feeling right now.”
catheterization B. “When the doctor arrives, everything will
B. Coronary artery bypass surgery be fine.”
C. "This is a bad situation, but you'll feel
better soon.”
D. “Please be assured we're doing A. Digoxin
everything we can to make you feel better.” B. Furosemide
C. Metoprolol
38. Which of the following classes of D. Enalapril
medications protects the ischemic
myocardium by blocking catecholamines 44. Which of the following symptoms is
and sympathetic nerve stimulation? A. most commonly associated with left-sided
Beta-adrenergic blockers hear failure? A. Crackles
B. Calcium channel blockers B. Arrhythmias
C. Narcotics C. Hepatic engorgement
D. Nitrates D. Hypotension
39. What is the most common complication
of a myocardial infarction? 45. In which of the following disorders
A. Cardiogenic shock would the nurse expect to assess sacral
B. Heart failure edema in bedridden client? A. DM
C. Arrhythmias B. Pulmonary emboli
D. Pericarditis C. Renal failure
D. Right-sided heart failure
40. With which of the following disorders is
jugular vein distention most prominent? 46. Which of the following symptoms might
A. Abdominal aortic aneurysm a client with right-sided heart failure exhibit?
B. Heart failure A. Adequate urine output
C. Myocardial infarction B. Polyuria
D. Pneumothorax C. Oliguria
D. Polydipsia
41. What position should the nurse place
the head of the bed in to obtain the most 47. Which of the following classes of
accurate reading of jugular vein distention? medications maximizes cardiac
A. High-Fowler's performance in clients with heart failure by
B. Raised 10 degrees increasing ventricular contractility? A.
C. Raised 30 degrees Beta-adrenergic blockers
D. Supine position B. Calcium channel blockers
C. Diuretics
42. Which of the following parameters D. Inotropic agents
should be checked before administering
digoxin? A. Apical pulse 48. Stimulation of the sympathetic
B. Blood pressure nervous system produces which of the
C. Radial pulse following responses? A. Bradycardia
D. Respiratory rate B. Tachycardia
C. Hypotension
43. Toxicity from which of the following D. Decreased myocardial contractility
medications may cause a client to see a
green halo around lights?
49. A 25-year-old male is admitted in sickle a chest tube inserted notes continuous
cell crisis. Which of the following gentle bubbling in the suction control
interventions would be of highest priority for chamber. What action is appropriate?
this client? A. Do nothing, because this is an
A. Taking hourly blood pressures with expected finding.
mechanical cuff B. Immediately clamp the chest tube
B. Encouraging fluid intake of at least and notify the physician
200mL per hour C. Check for an air leak because
C. Position in high fowlers with knee gatch the bubbling should be intermittent
raised D. Increase the suction pressure so
D. Administering Tylenol as ordered that bubbling becomes vigorous

50. A newly admitted client has 53. A nurse has assisted a physician
sickle cell crisis. The nurse is planning with the insertion of a chest tube. The
care based on assessment of the client. nurse monitors the adult client and
The client is complaining of severe pain notes fluctuation of the fluid level in the
in his feet and hands. The pulse water seal chamber after the tube is
oximetry is 92. Which of the following inserted. Based on the assessment,
interventions would be implemented which action would be appropriate?
first? Assume that there are orders for
each intervention: A. Adjust the room A. Inform the physician
temperature B. Continue to monitor the clien t C.
B. Give a bolus of IV fluids Reinforce the occlusice dressing.
C. Start Oxygen D. Encourage the client to deep-
D Administer meperidine breathe
(Demerol) 75mg IV push
54. The nurse is caring for a male
client with a chest tube, turns the client
51. A male client who takes
to the side and the chest tube
theophylline for chronic obstructive
accidentally disconnects. The initial
pulmonary disease is seen in the urgent
nursing action is to:
care center for respiratory distress.
A. Call the physician
Once the client is stabilized, the nurse
B. Place the tube in a bottle of
begins discharge teaching. The nurse
sterile water
would be especially vigilant to include
C. Immediately replace the chest
information about complying with
tube system
medication therapy If the client's baseline
D. Place the sterile dressing ove the
theophylline level was.
disconnection site.
A. 10 mcg/ml
B. 12 mcg/ml C. 15 mcg/ml D. 18
55. Nurse Paul is assisting a
mcg/ml
physician with the removal of a chest
tube. The nurse should instruct the client
52. Nurse Kim is caring for a client
to:
with a pneumothorax and who has had
A. Exhale slowly
B. Stay very still pulmonary disease Which of the
C. Inhale and exhale quickly following would the nurse expect to note
D. Perform the Valsalva maneuver on assessment of this client?
A. Hypocapnia
56. While changing the tapes on a B. A hyperinflated chest noted on
tracheostomy tube, the male client the chest ray
coughs and the tube is dislodged. The C. Increase oxygen saturation With
initial nursing acting is to: exercise
A. Call the physician to reinsert the D. A widened diaphragm noted on
tube the chest xray
B. Grasp the retention sutures to
spread the opening 60. A community health nurse is
C. Call the respiratory therapy conducting an educational session With
department to reinsert the community members regarding
tracheostomy tuberculosis. The nurse tells the group
D. Cover the tracheostomy wite with that one of the first symptoms
sterile dressing to prevent infection. associated with tuberculosis is:
A. Dyspnea
57. A nurse is caring for a male B. Chest pain
client immediately after removal of the C. A bloody, productive cough D.
endotracheal tube. The nurse reports A cough with the expectoration
which of the following signs immediately Of mucoid sputum
if experienced by the client?
A. Stridor 61 A nurse performs an
B. Occasional, pink-tinged sputum admission assessment on a female client
C. A few basilar lung crackles on With a diagnosis of tuberculosis.
the right The nurse reviews the results of
D. Respiratory rate of 24 breaths/min which diagnostic test that Will confirm this
diagnosis? A Bronchoscopy
58. An emergency room nurse is B Sputum culture
assessing a female client who has C Chest x-ray
sustained a blunt injury to the chest D Tuberculin skin test
wall. Which of these signs would
indicate the presence of a
62 The nursing instructor
pneumothorax in this client?
asks a nursing student to describe the
A. A low respiratory
route of transmission of tuberculosis. The
B. Diminished breathe sounds instructor concludes that the student
C. The presence of a barrel chest understands this Information if the
D. A sucking sound at the site of injury student states that the tuberculosis is
transmitted by:
59. A nurse is caring for a male A. Hand and mouth
client hospitalized with acute
B. The airborne route
exacerbation of chronic obstructive
C. The fecal-oral route
D. Blood and body fluids B Hematuria
C Bronchospasm
63. A nurse is caring for a male client D Blood-streaked sputum
with emphysema who is receiving
oxygen. The nurse assesses the oxygen 67. A nurse is suctioning fluids from a
flow rate to ensure that It does not male client via a tracheostomy tube
exceed. When suctioning, the nurse must
A. 1 L/min limit the suctioning time to a maximum
B 2 L/min of:
C 6 Limin A. 1 minute
D 10 L/min B. 5 seconds
C. 10 seconds
64. A nurse instructs a female client to D. 30 seconds
use the pursed-lip method of breathing
and the client asks the nurse about the 68. A nurse is suctioning fluids from a
purpose of this type of breathing. The female client through an endotracheal
nurse responds, knowing that the tube. During the suctioning procedure,
primary purpose of purSed-lip breathing is the nurse notes on the monitor that the
to: heart rate is decreasing. Which of the
A Promote oxygen intake. following is the appropriate nursing
B Strengthen the diaphragm. intervention?
C Strengthen the intercostal muscles. A Continue to suction.
D. Promote carbon dioxide B. Notify the physician immediately.
elimination. C. Stop the procedure and
reoxygenate the client.
65. Nurse Hannah is preparing to D. Ensure that the suction is limited
obtain a sputum specimen from a client. to 15 seconds
Which of the following nursing actions
will facilitate obtaining the specimen? 69. An unconscious male client is
A. Limiting fluids admitted to an emergency room. Arterial
B. Having the clients take three deep blood gas measurements reveal a pH of
breaths 7.30, a low bicarbonate level, a normal
C. Asking the client to split into the carbon dioxide level, a normal oxygen
collection container level, and an elevated potassium level.
D. Asking the client to obtain the These results indicate the presence of:
specimen after eating A. Metabolic acidosis
B. Respiratory acidosis
66 A nurse is caring for a female client C. Overcompensated respiratory
after a bronchoscopy and biopsy. Which acidosis
of the following signs, if noted in the D. Combined respiratory and
client, should be reported immediately to metabolic acidosis
the physicians?
A. Dry cough
70 A female client is suspected of C. Paradoxical respirations
having a pulmonary embolus. A nurse D. Pain, especially With inspiration
assesses the client, knowing that which
of the following is a common clinical 74 A female client with chest injury has
manifestation Of pulmonary embolism? suffered flail chest. A nurse assesses
A. Dyspnea the client for which most distinctive sign
B Bradypnea of flail chest
C Bradycardia A. Cyanosis
D Decreased respiratory B. Hypotension
C. Paradoxical chest movement
71. A nurse teaches a male client D. Dyspnea, especially on exhalation
about the use of a respiratory inhaler.
Which action by the client indicates a 75 A male client has been admitted
need for further teaching? with chest trauma after a motor vehicle
A. Inhales the mist and quickly accident and has undergone subsequent
exhales intubation. A nurse checks the client
B. Removes the cap and shakes the when the high-pressure alarm on the
inhaler well before use ventilator sounds, and notes that the
C. Presses the canister down with client has an absence of breath sounds
the finer as he breathes in in the right upper lobe of the lung. The
D. Waits 1 to 2 minutes between nurse immediately assesses for other
puffs if more than one puff has signs of: A Right pneumothorax
been B Pulmonary embolism
prescribed C Displaced endotracheal tube
72. A female client has just returned D Acute respiratory distress syndrome
to a nursing unit following
76. A client has just been diagnosed
bronchoscopy. A nurse would implement
with type 1 diabetes mellitus. Which
which Of the following nursing
comment by the client correlates best with
interventions for this client?
this disorder?
A. Administering atropine intravenously
A. "I was thirsty all the time. I just
B. Administering small doses of couldn't get enough to drink."
midazolam (Versed)
B. "It seemed like I had no appetite. I
C. Encouraging additional fluids for had to make myself eat."
the next 24 hours
C. "I had a cough and cold that just
D. Ensuring the return of the gag didn't seem to go away.'
reflex before offering food or fluids
D. "I noticed I had pain when I went to
the bathroom.
73. A nurse is assessing the respiratory
status of a male client who has suffered 77. Which outcome would indicate
a fractured rib. The nurse would expect successful treatment of diabetes insipidus?
to note which of the following? A. Slow
A. Fluid intake of less than 2,500 ml in 24
deep respirations hours
B. Rapid deep respirations B. Urine output of more than 200 ml/hr.
C. Blood pressure of 90/50 Trousseau's signs because they indicate:
D. pulse rate of 126 beats/minute A. Hypocalcemia
B. Hypercalcemia
79. A nurse administered NPH insulin C. Hypokalemia
to a client with Diabetes mellitus at 7 D. Hyperkalemia
am. At what time would the nurse
expect the client to be at greatest risk 83. On a medical-surgical floor, the
for a hypoglycemic reaction? A. 10 am nurse is caring for a cluster of clients who
B Noon have been diagnosed with diabetes
C. 4 pm mellitus. Which client should the nurse
D. 10 pm assess first?
A. An 80-year-old client with a blood
80. Which instruction should be glucose level of 350 mg/dl
included in the teaching plan for a client B. A 20-year-old client with a blood
requiring insulin? glucose level of 70 mg/dl
A. Administer insulin C. A 60-year-old client experiencing
B. Administer insulin at a 45 degree nausea and vomiting
angle into a deltoid muscle D. A 55-year-old client complaining
C. Shake the vial of insulin vigorously of chest pressure
before withdrawing the medication D.
Draw up clear insulin first when 84. The nurse is assigned to a 40-yr
mixing two types of insulin in one old client who has a diagnosis of
syringe chronic pancreatitis. The nurse reviews
81. A client with a serum glucose level the laboratory results, anticipating a
of 618 mg/dl is admitted to the facility. laboratory report that indicates a serum
He's awake and oriented; has hot, dry amylase level of:
skin; and has the following vital signs: A. 45 units/L
temperature 38.1 degrees Celsius, heart B. 100 units/L
rate of 116 beats/min and blood C. 300 units/L
pressure of 108/70 mmHg. Based on D. 500 units/L
these assessment findings, which
nursing diagnosis takes highest priority? 85. What laboratory finding is the
A. Deficient fluid volume related to primary diagnostic indicator for
osmotic diuresis pancreatitis? A. Elevated blood urea
B. Decreased cardiac output related to nitrogen (BUN)
osmotic diuresis B. Elevated serum lipase
C. Imbalanced Nutrition: less than body C. Elevated aspartate aminotransferase
requirements related to insulin deficiency (AST)
D. Ineffective thermoregulation related to D. Increased lactate dehydrogenase (LD)
dehydration
86. A nurse is caring for client with
82. For the first 72 hours after pheochromocytoma. The client asks for
thyroidectomy surgery, a nurse would a snack and something warm to drink.
assess a client for Chvostek's and
The most appropriate choice for this 90. A nurse is providing discharge
client to meet nutritional needs would be instructions to a client who has
which of the following? A. Crackers with Cushing's syndrome. Which client
cheese and tea statement indicates that instructions
B. Graham crackers and warm milk related to dietary management are
C. Toast with peanut butter and cocoa understood?
D. Vanilla wafers and coffee with cream and A. "I can eat foods that have a lot of
Sugar potassium in them."
B. "I will need to limit the amount of
87. A nurse is performing an protein in my diet."
assessment on a client with C. "I am fortunate that I can eat all the
pheochromocytoma. Which of the salty foods I enjoy."
following assessment data would indicate D. "I am fortunate that I do not
a potential complication associated with need to follow any special diet."
this disorder? A. A coagulation time of 5
minutes 91. A client is taking NPH insulin daily
B. A blood urea nitrogen level of 20 mg/dl every morning. The nurse instructs the
C. A urinary output of 50 ml per hour D. A client that the most likely time for a
heart rate that is 90 beats/min and hypoglycemic reaction to occur is: A. 2 to
irregular 4 hours after administration
B. 4 to 12 hours after administration
88. A nurse is preparing to provide C. 16 to 18 hours after administration
instructions to a client with Addison's D. 18 to 24 hours after administration
disease regarding diet therapy. The
nurse knows that which of the following 92. A nurse provides dietary
diets most likely would be prescribed for instructions to a client with diabetes
this client? mellitus regarding the prescribed diabetic
A. High fat intake diet. Which statement, if made by the
B. Low protein intake Client, indicates a need for further
C. Normal sodium intake teaching?
D. Low carbohydrate intake A"I need to drink diet soft drinks
B."I will eat a balanced meal plan.
89. A nurse is interviewing a client with C."I need to purchase special diabetic
type 2 diabetes mellitus. Which foods— sure ni?
statement by the client indicates an D"I'll snack on fruits instead of cake
understanding of the treatment for this
disorder? A. "I take oral insulin instead of 93. A client received 20 units of NPH
shots" B. "By taking these medications, insulin subcutaneously at 8:00 am. The
I am able to eat more." nurse should assess the client for a
C. "When I become ill, I need to hypoglycemic reaction at: A 10:00 am
increase the number of pills I take." B 11:00 am
D. "The medications I'm taking help C 5:00 pm
release the insulin I already make. D. 11:00 pm
94. The nurse is caring for a client Which of the following does the nurse
scheduled for a include in the teaching plan?
transsphenoidal hypophysectomy. The A. Exercise is best performed during
preoperative teaching instructions should peak times of insulin
include which most important statement? B. Administer insulin after exercising
A. *Your hair will need to be shaved." B. C. Take a blood glucose test before
Deep breathing and coughing will be exercising
needed after surgery.' D. Try to exercise prior to mealtime
C Brushing your teeth will not be
permitted for at least 2 weeks following 99. The nurse is preparing to
surgery. administer an IV insulin injection. The
D. You will receive spinal anesthesia. vial of regular insulin has been
refrigerated On inspection of the vial,
95. A nurse caring for a client with the nurse finds that the medication is
Addison's disease would expect to note frozen. The nurse should: A Wait for the
which of the following on assessment of insulin to thaw at room temperature
the client? A. Obesity B. check the temperature settings of the
B. Edema refrigerator
C. Hypotension D Hirsutism C Discard the insulin and obtain another
vial
96. A nurse is assessing a client with D Rotate the vial between the hands
a diagnosis of goiter. Which of the until the medication becomes liquid.
following would the nurse expect to note
during the assessment of the client? A 100. A nurse is assessing the learning
Client complains of slow wound healing readiness 'of a client newly diagnosed
B. Client complains of chronic fatigue with diabetes mellitus. Which client
C An enlarged thyroid gland behavior indicates to the nurse that the
D. The presence of heart damage client is not ready to learn?
A. The client complains of fatigue
97. The nurse is caring for a client following whenever the nurse plans a teaching
thyroidectomy. The nurse notes that session
calcium gluconate is prescribed for the B. The client asks if the spouse can
client. The nurse determines that this attend the teaching session
medication has been prescribed to: A. C. The client asks for written
Treat thyroid storm materials about diabetes mellitus before
B Prevent cardiac irritability class D. The client asks appropriate
C. Stimulate release of parathyroid questions about what will be taught
hormone D. Treat hypocalcemia tetany

98. The client with type 1 diabetes


mellitus is to begin an exercise program
and the nurse is providing instructions
to the client regarding the program.
NP 4 III. Talking
IV. Valsalva maneuver V.
1. Select the main structures below that Vomiting
play a role with altering intracranial VI. Keeping the head of the bed
pressure: between 30- 35 degrees

I. Brain
II. Neurons
III. Cerebrospinal Fluid
IV. Blood
V. Periosteum
VI. Dura mater

A. I, II, III
B. I, III, IV
C. III, IV, VI
D. All mentioned

2. The Monro-Kellie hypothesis explains the


compensatory relationship among the
structures in the skull that play a role with
intracranial pressure. Which of the following
are NOT compensatory mechanisms
performed by the body to decrease
intracranial pressure naturally? Select all
that apply:
I. Shifting cerebrospinal fluid to other
areas of the brain and spinal cord II.
Vasodilation of cerebral vessels III.
Decreasing cerebrospinal fluid production
IV. Leaking proteins into the brain barrier

A. land Il
B. land IV
C. ll and IV
D. Ill and IV

3.A patient is being treated for increased


intracranial pressure. Which activities below
should the patient avoid performing? Select
all that apply;
I. Coughing
II. Sneezing
A. I, II, IV,...........................................V
B. IlI IV. V C. I, IV. V.......................VI
D. II IV. V 4. A patient is experiencing hyperventilation and has a PaCO2 level of 52 The
patient has an ICP of 20 mmHg As the nurse you know that the PaCO2 level will? VI
D. Perform suctioning
A. cause vasoconstriction and decrease the
ICP 8.A patient has a ventriculostomy. Which
B. promote diuresis and decrease the ICP finding would you immediately report to the
C. cause vasodilation and increase the doctor?
ICP A. Temperature 98 4 'F
D. cause vasodilation and decrease the B. CPP 70 mmHg
ICP C. ICP 24 mmHg
D. PaC02 35
5. You're providing education to a
group of nursing students about ICP. You 9. External ventricular drains monitor IP and
explain that when cerebral perfusion are inserted where? A. Subarachnoid
pressure falls too low the brain is not space
properly perfused and brain tissue dies. A B. Lateral Ventricle
student asks, "What is a normal cerebral C. Epidural space
perfusion pressure level? Your response D. Right Ventricle
is:
A. 5-15 mmHq 10. Which of the following is contraindicated
B. 60-100 mmHg in a patient with increased ICP? A. Lumbar
C. 30-45 mmHg D. 160 mmHg puncture B. Midline position of the head C,
Hyperosmotis-diuretics.
6. Which patient below is at MOST D. Barbiturate medications
risk for increased intracranial pressure? A
patient who is experiencing severe 11. You’re collecting vital signs on a
hypotension. patient with ICP. The patient has a
B. A patient who is admitted with a Glascoma scale rating 4. How will you
traumatic brain injury. assess the patient's temperature? A.
C. A patient who recently experienced a Rectal
myocardial infarction B. Oral
D. A patient post op from eye surgery C. Axillary
7. A patient with increased ICP has the D. Auricle
following vital signs: blood pressure 99/60. 12. A patient who experienced a cerebral
HR 65, Temperature 101.6 'F, respirations hemorrhage is at risk for developing
14, oxygen saturation of 95%. IP reading is increased Which sign and symptom below
21 mmHg Based on these findings you is the EARLIEST indicator the patient is
would? having this complication? A. Bradycardia
A. Administered PR dose of a B. Decerebrate posturing
vasoresso! B. Administer 2 L Of oxvgen C. Restlessness
C. Remove extra blankets and give the D. Unequal pupil size
patient a cool patch
13. Select all the signs and symptoms that B. 74
occur with increased ICP C. 102
I. Decorticate posturing D. 88
II. Tachycardia
III. Decrease in pulse pressure 18. During the assessment of a patient with
IV. Cheyne-stokes increased ICP, you note that the patient's
V. Hemiplegia arms are extended straight out and toes
VI. Decerebrate posturing pointed downward. You will document this
as:
A. I, II, III, IV A. Decorticate posturing
B. I, IV. V. VI B. Decerebrate posturing
C. III, IV, V, VI C. Flaccid posturing
D. All mentioned D. Catatonia

14. You're maintaining an external 19. While positioning a patient in bed with
ventricular drain The ICP readings should increased ICP, it important to avoid? A.
be? Midline positioning of the head
A. 5 to 15 mmHg B. Placing the HOB at 30-35 degrees
B. 20 to 35 mmHg C. Preventing flexion of the neck
C. 60 to 100 mmHg D. Flexion of the hips
D. 5 to 25 mmHg
20. During the eye assessment of a patient
15. Which patient below with ICP is with increased ICP, you need to assess tha
experiencing Cushing's Triad? A patient oculocephalic reflex. If the patient has brain
with the following: stem damage what response will you find?
A. BP 150/112, HR 110, RR 8 A. The eyes will move in the same direction
B. BP 90/60. HR 80. RR 22 as the head is moved side to side B. The
C. BP 200/60, HR 50, RR 8 eyes will move in the opposite direction as
D. BP 80/40, HR 49, RR 12 the head is moved side to side C. The eves
will roll back as the head is moved side to
16. The patient has a blood pressure of side.
130/88 and ICP reading of 12. What is the D. The eyes will be in a fixed midline
patient's cerebral pension pressure, and position as the head is moved side to side.
now do you interpret this as the nurse? 21. All the following causes of Spinal Cord
A. 90 mmHg, normal Injuries are non-traumatic in nature, which is
B. 62 mmHg, abnormal not included?
C. 36 mmHg, abnormal A. Rheumatoid Arthritis and Ankylosing
D. 56 mmHa, normal B. Spondylitis
C. Vascular problems
17. Per question 16, the patient's blood D. Electric shock
pressure is 130/88. What is the patient's
mean arterial pressure (MAP)? 22. Mr. Yoshihiro Sato, an Olympic
A. 42 swimmer, suffered from a diving accident
and had respiratory arrest before being
transferred to the hospital. The nurse C. Increased appetite
seeing the scene opens the patient's D. Weight gain
airway to provide rescue breathing using
which maneuver? 27. A nurse is caring for a client with a
A. Head tilt diagnosis of gout. Which of the following
B. Jaw thrust laboratory values would the nurse expect to
C. Chin lift note in the client
D. Logroll technique A Calcium level of 9.0 mg/dL
B. Uric acid level of 8.6 mg/dL
23. Injuries involving the spinal cord in C. Potassium level of 4.1 mg/dL
the thoracic level will lead to a paralysis D. Phosphorus level 3.1 mg/dL
confined to the lower limbs, a condition
known as A.Aletradlecia 28. A nurse is caring for a client with
B.Hemiplegia osteoarthritis . The nurse performs an
C.Quadriplegia assessment, knowing that which of
D.Paraplegia the following is a clinical
manifestation associated with the
24. Mr. Blake is confirmed to be having disorder? A. Morning stiffness
a spinal cord injury at the sacral level (S3). B. A decreased sedimentation rate
The nurse includes in the plan of care of C. Joint pain that diminishes after rest D.
Mr. Blake, which interventions? Elevated antinuclear antibody levels
A Insertion of a foley catheter B.Monitoring
the patient while being hooked to a 29. The client has had surgery to repair a
mechanical ventilator fractured hip. The nurse obtains which of
C. Exercises to prevent atrophy of the the following most important items from the
paralyzed upper and lower extremities unit storage area to use when repositioning
D.Coping strategies for sensory and motor the client from side to side in bed?
deficits on the left/right half of the body. A. Abductor splint
E All the above B. Adductor splint
25. Which of the ff. is given to a patient C. Bed pillow
with spinal cord injury, primarily to address D. Overhead trapeze
hypotension?
A.Calcium channel blocker 30. The nurse has developed a plan of care
B.Dextran for a client who is in traction and documents
C.Methylprednisolone Sodium Succinate a nursing diagnosis of self-care deficit. The
D.Mannitol nurse evaluates the plan of care and
determines which of the following
26. A nurse is reviewing the health care observations indicates a successful
record of a client with a new diagnosis of outcome?
rheumatoid arthritis (RA). The nurse A. The client allows the nurse the nurse to
understands that which of the following is complete the care daily
an early clinical manifestation of RA? A. B. The client allows the family to assist in
Complaints of fatigue the care
B. Increased energy level C. The client refuses care
D. The client assists in self-care as much A. Impaired arterial circulation
as possible B. The presence of an infection
C. Impaired venous return
31. A home care nurse is visiting a client D. Arterial insufficiency
who is in a body cast. The nurse is
performing an assessment and Is assessing 35. A client is complaining of knee pain.
the psychosocial adjustment of the client to The knee is swollen, reddened and warm to
the cast. The nurse would most touch. The nurse interprets that the client's
appropriately assess the signs and symptoms are not compatible
A. Type of transportation available for with:
follow-up care A. Inflammation
B. Ability to perform activities of daily living B. Degenerative disease
C. Need for sensory stimulation C. Infection
D. Amount of home care support available D. Recent injury

32. A community health nurse is providing 36. A nurse witnesses a client sustain a fall
an educational session for community and suspects that the right leg may be
members regarding dietary measures that broken. The nurse takes which priority
will assist in reducing the risk of action?
osteoporosis. The nurse instructs the A. Take a set of vital signs
community members to increase dietary B. Call the radiology department C.
intake of which food known to be helpful Reassure the client that everything will
in minimizing this risk? be
A. Yogurt D. Immobilize the right leg before moving
B. Turkey the client.
C. Spaghetti 37. A nurse in the hospital emergency
D. Shellfish department is caring for a client with a
33. A nurse is teaching a client with a right fractured arm and is preparing the client for
arm cast how to prevent stiff or frozen a reduction of the fracture that will be done
shoulder. The nurse should instruct the in the casting room. Which of the following
client to: is unnecessary?
A Keep a sling on the arm always A. Explanation of the procedure to the client
B.Lift the shoulder of the casted arm B. Administration of an analgesIc
over the head-periodically throughout C. Anesthesia consent
the day D. Consent for the procedure
C. Avoid range-of-mation exercises
D. Wear the sling at nighttime 38. A nurse has suggested specific leg
exercises for a client immobilized in right
34. A nurse is performing neurovascular skeletal lower leg traction. The nurse
assessment on a client with a cast on the determines that the client needs further
left lower leg. The nurse notes the instruction of the nurse observes the client:
presence of edema in the foot below the A. Pulling up using the trapeze
cast. The nurse would interpret that this
finding indicates:
B. Flexing and extending the feet C. B. Chest radiograph
Performing active range of motion to the C. Tetanus vaccine
right ankle and knee D. Tuberculin test
D. Doing quadriceps-setting and gluteal
setting exercises 43. A client has just been admitted to the
hospital with a fractured femur and pelvic
39. A client has a slight weakness in the fractures. The nurse plans to carefully
right leg. Based on this assessment finding, monitor the client for which of the
the nurse determines that the client would following signs and symptoms? A.
benefit most from the use of A. walker Tachycardia, hypotension
B. A wooden crutch B. Bradycardia, hypertension
C. A Lofstrand crutch C. Fever, bradycardia
D. A straight leg cranes D. Fever, hypertension

40. A client who has experienced a brain 44. A client is complaining of pain
attack (stroke) has partial hemiplegia of the underneath a cast in a bony prominence.
left leg. The straight leg cane formerly used The nurse interprets that this client may
by the client is not quite sufficient now. The need:
nurse interprets that the client could benefit A. To have the cast replaced with an air
from the somewhat greater support and splint
stability provided by a B. To have extra padding put over this area
A. Quad cane of the cast
B. Wooden crutch C. To have the cast bivalve
C. Loistrand crutch D. To have a window cut in the cast
D. Wheelchair 45. A test for the presence of rheumatoid
41. A client who is learning to use a cane is factor is performed in a client with a
afraid that they will slip with ambulation, diagnosis of rheumatoid arthritis. The nurse
causing a fall. The nurse provides the client understands that this test assesses for the
with the greatest reassurance by stating presence of:
that. A. Unusual antibodies of the IgG and IgM
A. Canes prevent falls; they do not cause type B. Antigens
ther of IgA
B. The cane has a flared tip with C. Inflammation
concentric rings to give stability C. The D. Infection in the body
physical therapist will determine if the
cane is inadequate 46. You are initiating a nursing care plan
D. The cane would help to break a fall, for a patient with osteoporosis. These
even if you do slip nursing interventions apply to the nursing
diagnosis Risk for Falls. Which intervention
42. A nurse in the hospital emergency should you delegate to the nursing
department is assessing a client with an assistant? A. Identify environmental factors
open leg fracture. The nurse inquires that increase risk for falls.
about the date of the client's last A. B. Monitor gait, balance, and fatigue
Physical examination level with ambulation.
C. Collaborate with physical therapy to 50. As charge nurse you are making
provide patient with walker assignments for the day shift. Which patient
D. Assist the patient with ambulation to would you assign to the nurse who has
bathroom and in halls. been pulled from the post-anesthesia care
unit (PACU) for the day?
47. You are preparing to teach a newly A. A 35-year-old patient with
diagnosed patient with osteoporosis about osteomyelitis who needs teaching prior to
strategies to prevent falls. Which of these hyperbaric oxygen therapy
points will you be sure to include? (Choose B. A 62-year-old patient with
all that apply.) osteomalacia who is being discharged to a
A. Wear a hip protector when long-term care
ambulating.- facility
B. Remove throw rugs and other C. A 68-year-Old patient with
obstacles at home- osteoporosis and a new orthotic device
C. Exercise will help build your whose knowledge of use of this device
strength- D. You should expect a few must be assessed. D. A 72-year-old patient
bumps and bruises when you go home E. with Paget's disease who has just returned
When you are tired, you should rest.- from surgery for total knee replacement

48. You discover these assessment 51. A patient is admitted to the hospital
findings when admitting a patient with after vomiting bright red blood and is
Paget's disease. Which finding indicates diagnosed with a bleeding duodenal ulcer.
that the physician should be notified? The patient develops a sudden, sharp pain
A. A. The patient has bowing of both legs in the midepigastric region along with a
and the k nees are asymmetricasymmetric rigid, boardlike abdomen. These clinical
B. The base of the patient's skull is manitestations most likely indicate which of
Invaginated (platypasia). the following?
C. The patient is only 5 feet tall and A. An intestinal obstruction has developed.
weighs 120 pounds. B. The ulcer has perforated.
D. D. The patient's skull is soft, thick, C. Additional ulcers have developed
and larger than normal D. The esophagus has become inflamed

49. As charge nurse you observe the 52. The patient asks the nurse what causes
LPN/LVN providing these interventions for a Peptic Ulcer to develop. The nurse
the patient with Paget's disease. Which responds that recent research indicates that
action requires that you intervene? many peptic ulcers are the result of which of
A. Administers 600 mg of ibuprofen to the following?
the patient A. Helicobacter pylori infection
B. Encourages the patient to perform B. Diets high in fat
PT recommended exercises C. Work-related stress.
C. Applies ice and gentle massage to D. A genetic defect in the gastric mucosa.
the patient's lower extremities
D. Reminds the patient to drink milk 53. The nurse is preparing to teach a
and eat cottage cheese patient with a Peptic Ulcer about the diet
that should be followed by discharge. The 57. After a subtotal Gastrectomy, the
nurse should explain that the diet will most nurse should anticipate that NGT drainage
likely consist of the following. will be what color for about 12 to 24 hours
A. Any foods that are tolerated. after surgery?
B. Large amounts of milk. A. Bright red.
C. Bland foods. B. Cloudy white.
D. High-protein foods. C. Dark brown.
D. Bile green.
54. A patient is to take one daily dose of
ranitidine (Zantac) at home to treat her 58. The nurse understands that the best
peptic ulcer. The nurse knows that the position for the patient who has undergone
patient understands proper drug a Gastrectomy is: A. Supine.
administration of ranitidine when she says B. Prone.
that she will take the drug at which of the C. Low Fowlers
following times? D. Right or left Sim's.
A. When pain occurs.
B. Before meals. 59. To reduce the risk of dumping
C. With meals. syndrome, the nurse should teach the
D. At Bedtime patient which of the following interventions?
A. Decrease the carbohydrate content of
55. Which of the following would be an meals.
expected outcome for a patient with Peptic B. Avoid milk and other dairy products.
Ulcer disease? C. Drink liquids with meals, avoiding
A. The patient will explain the rationale caffeine
for eliminating alcohol from the diet . D. Sit upright for 30 minutes after meals.
B. The patient will verbalize the
importance of monitoring hemoglobin and 60. Which of the following symptoms would
hematocrit every 3 months. be indicative of the Dumping Syndrome?
C. The patient will demonstrate A. Diaphoresis.
appropriate use of analgesics to control B. Vomiting.
pain. C. Hunger D. Heartburn.
D. The patient will eliminate contact
sports from his or her lifestyle 61. After surgery for gastric cancer, a
patient is scheduled to undergo radiation
56. A patient with suspected gastric cancer therapy. It will be most important for the
undergoes an endoscopy of the stomach. nurse to include information about which of
Which of the following assessments made the following in the patient's teaching plan?
after the procedure would indicate the A. Nutritional intake B. Exercise and
development of a potential complication? A. activity levels.
The patient complains of a sore throat. B. C. Management of alopecia.
The patient demonstrates a lack of appetite D. Access to community resources.
C. The patient displays signs of sedation.
D. The patient experiences a sudden
increase in temperature
Situation: A patient who has been B. Urinary tract infection.
diagnosed with Gastroesophageal reflux C. Prolonged bleeding time.
disease (GERD) complains of heartburn. D. Respiratory distress.**

62. To decrease the heartburn, the 67. How much bile would the nurse
nurse should instruct the patient to expect the T- tube to drain during the first
eliminate which of the following items from 24 hours after a Choledocholithotomy? A.
the diet? A. Hot chocolate. 300 to 500 mL.
A. Air-popped popcorn. B. 550 to 700 mL.
B. Raw vegetables C. 50 to 100 mL.
C. Lean beef D. 150 to 250 mL

63. The patient with GERD complains of 68. After a Cholecystectomy it is


a chronic cough. The nurse understands recommended that the patient follow a low-
that in a patient with GERD this symptom fat diet at home. Which of the following
may be indicative of which of the following foods would be most appropriate to include
conditions. in a low-fat diet?
A. Aspiration of gastric contents . A. Roast beef.
B. Development of laryngeal cancer. B. Cheese omelet
C. Esophageal scar tissue formation. C. Peanut butter
D. Irritation of the esophagus. D. Ham salad sandwich
64. The patient attends two sessions
with the dietician to learn about diet 69. Which of the following discharge
modifications to minimize instructions would be appropriate for a
Gastroesophageal Reflux. The teaching patient who has had a laparoscopic
would be considered successful if the Cholecystectomy?
patient says that she will decrease her A. Change the dressing daily until the
intake of which of the following foods? incision nedis
A. Fats B. High-sodium foods ( Leave dressings in place until you
C. Carbohydrates. see the surgeon at the postoperative
D. High-calcium foods. visit. Sa quizlet)
B. Avoid showering for 48 hours after
65. Which position would be ideal for the surgery.
patient in the early postoperative period C. Use acetaminophen (Tylenol) to control
after a Hemorrhoidectomy? A. Supine. any fever
B. Side-lying. D. Return to work within 1 week.
C. Trendelenburg
D. High-Fowler's. 70. Celso is admitted to the hospital
with acute pancreatitis. The nurse
66. When the patient's common bile duct taking a history should question the
is obstructed, the nurse should evaluate client about which of these risks for
the patient for signs of which of the developing pancreatitis ? A.
following complications? inflammatory bowel disease
A. Circulatory overload.--?
D. alcoholism Situation: Pedro consulted a urologist
B. diabetes mellitus because of his chronic renal problem. The
D. high-fiber diet physician advised him to undergo peritoneal
dialysis.
71. Which of the following factors should be
the focus of nursing management in a client 76. Which of the following assessments
with Acute Pancreatitis? A. Fluid and would be most appropriate for the nurse to
electrolyte balance.- make while the dialysis solution is
B. Management of hypoglycemia dwelling within the patient's abdomen?
C. Pain control. A. Check capillary refill time
D.Dietary management. B. Assess for urticaria.
C. Monitor electrolyte status D.
72. In alcohol-related pancreatitis, which of Observe respiratory status.
the following interventions is the best way to
reduce the exacerbation of pain? A. Eating 77. The dialysis solution is warmed before
a low-fat diet. use in peritoneal dialysis primarily to:
B. Abstaining from alcohol. A. force potassium back into the cells
C. Lying supine. B. promote abdominal muscle
D. Taking aspirin. relaxation.
73. Which of the following findings would C. add extra warmth to the body
strongly indicate the possibility of Cirrhosis? D. encourage the removal of serum urea
A. Pruritus B.
Peripheral edema. 78. During dialysis, the nurse observes that
C. Hepatomegaly. the flow of dialysate stops before all the
D. Dry skin. solution has drained out. The nurse should:
A. Reposition the peritoneal catheter
74. The nurse is aware that the B. Have the patient sit in a chair.
symptoms of Portal Hypertension in C. Have the patient walk.
clients with liver cirrhosis are chiefly the D. Turn the patient from side to side.
result of. A. Infection of the liver
parenchyma 79. Which of the following nursing
B. Fatty degeneration of Kupffer cell interventions should be included in the
C. Obstruction of the portal circulation D. patient's care plan during dialysis therapy?
Obstruction of the cystic and hepatic A. Keep the patient NPO.
ducts B. Monitor patient's blood pressure.
C. Limit the patient's visitors.
75. Which goal for the patient's care should D. Pad the side rails of the bed.
take priority during the first day of
hospitalization for an exacerbation of 80. What is the most potentially
Ulcerative Colitis? dangerous complication of peritoneal
A. Maintaining adequate nutrition. dialysis? A. Muscle cramps.
B. Managing diarrhea. B. Abdominal pain.
C. Promoting self-care and independence. C. Gastrointestinal bleeding.
D. Promoting rest and comfort. D. Peritonitis
81. After completion of peritoneal 85. Which of the following symptoms
dialysis, the nurse would expect the would the nurse most likely expect the
patient to exhibit which of the following patient to report during the assessment?
characteristics? A. Weight loss. A. Hematuria
B. Hypertension. B. Frequency and burning on urination
C. Hematuria. C. Flank pain and nausea
D. Increased urine output. D. Fever and chills

Situation: Cecil a 38-year married woman, 86. The patient asks the nurse, "How did
seeks consultation for painful urination, I get this urinary tract infection?" The nurse
urgency in voiding and fever for 3 days. should explain that in most instances,
cystitis is caused
82. Which of the following symptoms A. Congenital strictures in the urethra.
would most likely indicate Pyelonephritis? B. An infection elsewhere in the body
A. Costovertebral angle (CVA) tenderness C. Urine stasis in the urinary bladder D. An
B Nausea and vomiting ascending infection from the urethra.
C. Ascites 87. The patient is afraid to discuss her
D. Polyuria. diagnosis of Cystitis with her husband.
83. The nurse is aware that one of the Which would be the nurse's best approach?
following laboratory values will support a A. Spend time with the patient addressing
diagnosis of Pyelonephritis. her concerns and then stay with her while
A. Myoglobinuria she talks with her husband.
B. Pyuria B. Talk first with the husband alone
C. Ketonuria and then with both of them together to
D. Low white blood cell (WBC) count share the husband's reactions.
C. Insist that the patient talk with her
84. The patient with acute Pyelonephritis husband because good communication is
wants to know the possibility of developing necessary for a successful marriage D.
chronic Pyelonephritis. The nurse's Arrange a meeting with the patient, her
response is based on knowledge that which husband the doctor and the nurse
of the following disorders most commonly
leads to chronic Pyelonephritis? 88. The nurse teaches a patient some
A. Acute renal failure method to relieve her discomfort until the
B. Recurrent UTI C. Acute antibiotic takes effect. Which of the following
pyelonephritis. responses by the patient would indicate that
D. Glomerulonephritis. she understands the Nurse's instructions?
A. "I will place ice packs on my perineum."
Situation: Gina 24-year-old, patient who is B. I will drink a cup of warm tea every
newlywed comes to an ambulatory clinic in hour
moderate distress with a diagnosis of acute C. *I will take hot tub baths." D. I will
cystitis, void every 5 to 6 hours.

89. Which of the following statements by


the patient would indicate that she is at high
risk for recurrence of Cystitis? A. "I drink a B. breathe deeply
lot of water during the day C. Turn to the side
B. "I take a tub bath every evening C. "I D. hold the labia or shaft of penis.
wipe from front to back after voiding.
D. "I can usually go 8 to 10 hours without 94. Which steps should the nurse follow to
needing to empty my bladder. insert a straight urinary catheter?
A. Create a sterile field, drape client, clean
90. A client who has been treated for meatus, and insert catheter only 6 B. Put
chronic renal failure (CRF) is ready for on gloves, prepare equipment, create a
discharge. The nurse should reinforce which sterile field, expose urinary meatus, and
dietary instruction? A. "Be sure to eat meat insert catheter 6"
at every meal" B. "Monitor your fruit intake, C. Prepare client and equipment,
and eat plenty of bananas. Create à sterile field, put on gloves, clean
C. "Increase your carbohydrate intake urinary meatus, and insert catheter until
D. "Drink plenty of fluids, and use a salt urine flows.
substitute." D. Prepare client and equipment, sterile
field test catheter balloon clean meals, and
91. A client with renal failure is undergoing insert catheter until unne flows
continuous ambulatory peritoneal dialysis.
Which nursing diagnosis is most appropriate 95. Which of the following is an appropriate
for this client? nursing diagnosis for a client with renal
A. Altered urinary elimination calculi?
B. Toileting self-care deficit A.Ineffective tissue perfusion
C. Risk for infection B. Functional urinary incontinence
D. Activity intolerance C.Risk for infection
D. Decreased cardiac output
92. The client underwent a transurethral
resection of the prostate gland 24 hours ago 96. Which clinical manifestation would
and is on continuous bladder irrigation. lead the nurse to suspect that a client Is
Which of the following nursing interventions experiencing hypermagnesemia? A.
is appropriate? Muscle pain and acute rhabdomyolysis
A. Tell the client to try to urinate around B. Hot, flushed skin and diaphoresis
the catheter to remove blood clots B. C.Soft-tissue calcification and hyperreflexia
Restrict fluids to prevent the client's D. Increased respiratory rate and depth
bladder from becoming distended. C.
Prepare to remove the catheter. 97. Joshua is receiving furosemide and
D. Use aseptic technique when irrigating Digoxin, which laboratory data would be the
the catheter most important to assess in planning the
care for the client? A. Sodium level
93. The nurse is inserting a urinary catheter B. Magnesium level
into a client who is extremely anxious about C. Potassium level
the procedure. The nurse can facilitate the D. Calcium level
insertion by asking the client to:
A. initiate a stream of urine
98. Mr. Salcedo has the following arterial
blood gas (ABG) values: pH of 7.34, partial
pressure of arterial oxygen of 80 mm Hg,
partial pressure of arterial carbon dioxide of
49 mm Hg, and a bicarbonate level of 24
mEq/L. Based on these results, which
intervention should the nurse implement?
A. Instructing the client to breathe slowly
into a paper bag B. Administering low-flow
oxygen C. Encouraging the client to
cough and deep breathe
D. Nothing, because these ABG values are
within normal limits.
99. A client is diagnosed with metabolic
acidosis, which would the nurse expect the
health care provider to order?
A. Potassium
B. Sodium bicarbonate
C. Serum sodium level
D. Bronchodilator

100. Lee Angela's lab test just revealed


that her chloride level is 96 mEq/L. As a
nurse, you would interpret this serum
chloride level as: A. high
B. low
C. within normal range
D. high normal
NP 5 4. A client with impaired vision is admitted
to the hospital. The following interventions
1. A patient who has a hemorrhage in the are appropriate to meet the client's needs
vitreous cavity of the eye, the nurse EXCEPT
knows that blood is accumulating A. in A. Identify yourself by name B.
the aqueous humor. Stay in the client's field of vision.
B. between the lens and the retina C. Explain the sounds in the environment
C. between the cornea and the lens. D. Decrease background noise before
D. in the space between the iris and the speaking.
lens.
5. The nurse should always assess the
2. The nurse instructs a family member patient with an ophthalmic problem for
how to guide a visually impaired person A. Visual acuity.
when ambulating by: B. pupillary reactions
A. holding the visually impaired person C. intraocular pressure
by his or her nondominant arm and walking D. confrontation visual fields
side by side.
B. holding the nondominant hand, 6. Increased intraocular pressure may
wrapping the arm around his or her waist, occur as a result of
and walking side by side. A. edema of the corneal stroma
C. allowing the visually impaired person B. dilation of the retinal arterioles.
to hold the helper's arm, with the helper C. blockage of the lacrimal canals and
slightly ahead. ducts.
D. allowing the visually impaired person D. increased production of aqueous humor
to hold the shoulder of the helper and walk by the ciliary body.
slightly behind the helper.
7. A patient is diagnosed with close-angle
3. The patient tells you that he has to hold glaucoma. The nurse prepares the patient
his paper farther and farther away from his on what diagnostic procedure to visualize
face to read it. It has become a joke in his the anterior chamber angle?
family about how far away he needs to hold A. Ophthalmoscopy
reading material. You tell the patient: B. Gonioscopy
A. "You have myopia. Glasses will help C. Retinoscopy
you read.” D. Bio microscopy
B. You may have astigmatism, and
your eyes will get used to the problem. 8. The nurse should specifically question
C. "You have presbyopia, which is a patients using eyedrops to treat glaucoma
normal age-related change. Reading about
glasses will help you. A. use of corrective lenses.
D. "You may have an eye infection that B. their usual sleep patterns
is affecting your vision. You will need an C. a history of heart or lung disease.
antibiotic ointment to instill into your eyes D. sensitivity to opioids or depressants.
9. When assessing the visual fields in treatment because of severely diminished
acute glaucoma, the nurse would expect to hearing acuity. His ears were inspected by
find a: A. clear cornea. an Otolaryngologist and it was found Out
B. marked blurring of vision. that both of his auditory canals were
C. constricted pupil. obstructed by an impacted cerumen.
D. watery ocular discharge
13. Nurse Savannah is knowledgeable
10. A client is diagnosed with acute angle that Mr. Harris' condition is most
glaucoma and is placed on oral osmotic commonly caused by: A. an acute ear
diuretic therapy . Which of the following will infection.
the Nurse include in his health teachings B. overactive ceruminous glands
during acute phase of his illness? C. poor hygiene.
A. Measurement of intake and output. D. Concurrent dermatosis
B. Using metal eye patch to protect the eye.
C. Otic drop administration. 14. Initial intervention for an impacted
D. Keeping his belonging well organized at cerumen involves irrigation of a
all times. -? cerumenolytic agent to the involved ear.
Nurse Savannah prepares which solution to
11. When irrigating a patient's ear, the most effectively soften the impacted
nurse will: straighten the ear canal and cerumen? A. 0.9% NaCI solution
irrigate with a large-tipped bulb syringe. B. Gentamycin drops
A. Straighten the ear canal and irrigate with C. Atropine Sulfate drops
a large-tipped bulb syringe D. Peroxide in Glyceryl
B. direct the solution to the middle of the
canal to avoid damaging the ear. 15. When the nurse reads in the patient's
C. use a body temperature solution and history that the patient has experienced
have the patient hold a basin under the otalgia , the nurse knows that the patient
ear while directing the solution toward has:
the top of the canal. A. difficulty hearing.
D. repeat the irrigation with hotter water. B. a buildup of cerumen.
C. ear pain.
12. Nurse Sylvia was approached by a D. ringing in ears.
teenager crying in pain because of an ant
that has entered her ears. What should be 16. Nursing instructions for a patient
Nurse Sylvia's action? suffering from external otitis should include
A. Irrigate the ear using a cerumenolytic the
agent. A. Application of heat to the auricle.
B. Use a moist cotton bud to remove the B. Avoidance of swimming.
insect. C. Ingestion of over-the-counter analgesics,
C. Instill a small amount of mineral oil such as aspirin.
D. Blow a smoke towards the patient's ear, D. All of the above.

Situation: Mr. Timothy Harris, a 78-year-old 17. The nurse points out to the client that of
patient, subjected himself to medical the activities in which he is regularly
involved, the one that contributes to D. Follows a forceful nose blowing
accumulation of cerumen in the external ear Situation: Mr. Tonio a 49-year-old, a jolly
is: person works at the Commercial bank for
A. swimming in a chlorinated pool daily. 13 years had been informed for some
B. drinking 1500 mL. of fluid a day. changes in is work load.m Three
C. trimming hair from the ears every week months after he was observed of
D. washing the ears with a washcloth unbecoming behavior. He was
every day. diagnosed with Bipolar I disorder and
experiencing a Manic episode is newly
Situation: Otitis media is currently admitted to the inpatient psychiatric unit.
considered as an important public health
problem which may produce long-term 21. Which nursing diagnosis is a priority at
effects on language, auditory and this time?
cognitive development. A. Risk for violence: other-directed R/T poor
impulse control
18. Which classification of otitis media B. Altered though process RT hallucinations
according to duration is APPROPRIATELY C. Social isolation R/T manic excitement D.
described? Low self-esteem R/T guilt about
A Acute Otitis Media = resolves within 6 promiscuity
months
B. Chronic Otitis Media = occurs longer 22. In the mental hospital Mr. Tonio is
than yelling at another peer in the milieu. Which
12 months nursing intervention takes priority?
C. Subacute Otitis Media = occurs for 6 to A. calmly redirect and remove the client
12 months from the milieu
D. All of the above B. administer prescribed PRN
E. None of the above intramuscular injection for agitation C.
notify the client to lower voice D. obtain an
19. Identification of risk factors in key to order for seclusion to help decrease
prevention of Otitis Media. The nurse lists external stimuli
down the risk factors and includes which of
the following? (Select all that apply) 23. Another client diagnosed with bipolar Il
A.Chronic upper respiratory infection disorder has a nursing djagnosis of
B.Congenital abnormalities (Cleft Palate impaired social interactions R/T
Down Syndrome) egocentrism. Which short-term outcome is
C.Daily intake of aspirin an appropriate expectation for this client
D. Exposure to secondhand smoke problem?
A. the client will have an appropriate
20. Suppurative Otitis media can be due one-on- one interaction with a peer by
to the following BUT ONE A. May follow day 4
a viral disease. B. the client will exchange personal
B. Barotrauma. information with peers at lunchtime C.
C. Tympanic membrane perforation the client will verbalize the desire to
interact with peers by day 2 D. the client
will initiate an appropriate social B. The appropriateness of the interview
relationship with a peer questions posed
24. An adult client 52 years old, diagnosed C. Faithfulness to everyday reality of the
with major depressive disorder is being participants
considered for electroconvulsive therapy D. The adequacy of the coding system used
(ECT). Which client teaching should the
nurse prioritize? 27. What is a characteristic of an intrinsic
A. empathize with the client about fears case study?
regarding ECT A. It yields a better understanding of each
B. monitor for any cardiac alterations to case.
avoid possible negative outcomes C. B. It provides a foundation to challenge a
discuss the client and family expected generalization.
short-term memory loss C. It does not include quantitative data.
D. inform the client that injury related to D. It can scrutinize only uncomplicated
induced seizure commonly occurs Phenomena.

25. A nursing instructor is teaching about 28. Which of the following is most
the cause of mood disorders. Which accurate regarding the grounded-theory
statement by a nursing student best method? A. Data are collected using an
indicates an understanding of the etiology of etic perspective.
mood disorders? B. It is a process of constructing human
A. When clients experience loss, they experience.
learn that it is inevitable and become C. Secondary sources are sometires used
hopeless and helpless. D. It is an inductive approach.
B. "There are alterations in the
neurochemicals, such as serotonin, which 29. Nursing Research can be classified
cause the client's symptoms C. "Evidence according to the time frame the research
continues to support multiple causations study has been made As a nurse, you know
related to an individual's susceptibility to that the study entitled *Knowledge and
mood symptoms.' practice of Staff Nurses in preventing
D. "There is a genetic component affecting needle prick injuries in a private hospital in
the development of mood disorder." Manila is classified as
A. Basic Research
Situation: Paulo 9 years old is diagnosed B. Historical Research
with an autistic disorder makes no eye C. Descriptive Research
contact; is unresponsive to staff members D. Experimental Research
and continuously twists, spins, and head
bangs, 30. Nursing Research has a lot of
purposes, Which of the following category
26. What is meant by the "fittingness" of a do the study about sociodemographic
research study? profile of nursing students and their risk for
A. Truth of findings as judged by the depression belong? A. Prescription
participants B. Exploration
C. Prediction and control
D. Explanation 34. Based on the research title which of the
following will be the general objective?
31. In an experimental research, as a nurse A. This study aims to determine the
there must be an essential aclivily wherein level of knowledge and practice of
the participants of the study will be able to prevention of needle prick injuries of Staff
understand the whole experimental design. Nurses in a certain private hospital.
This term is called: B. This study aims to determine if there
A. Desensitization is a significant relationship between levels
B. Experimental Proper of knowledge regarding prevention of
C. Research Proper needle prick injuries and sociodemographic
D. Debriefing of Staff nurses in a certain private hospital.
C. The study aims to determine the
32. Ethics in Nursing Research has always levels of knowledge regarding prevention
been an issue when it comes to the identity of needle prick injuries of staff nurses in a
of the respondents. When the topics of certain private hospital
research are very sensitive, which of the D. This study aims to determine the
following rights of individual participants isociodemographic of staff nurses in a
must be ensured when the researcher certain private hospital.
cannot link the information given by the
respondent from the source of the 35 All of these are specific objectives
information? A. Confidentiality except A. This study is for nursing
B. Anonymity researchers in the future.
C. Virility B. This study aims to determine if there is a
D. Volunteerism significant relationship between levels of
knowledge regarding prevention of needle
33. In starting a focused group discussion, prick injunes and sociodemographic of Staff
Nurse Dina wants to stress out the Nurses in a certain private hospital.
confidentiality of the topics that they will be C.This study aims to determine the levels
discussing. Which of the following of knowledge regarding prevention of
instruction convey confidentiality? needle prick injuries of Staff Nurses in a
A. "This discussion should not only be certain private hospital.
confined within this group of people. Any D. This study aims to determine the
information discussed should be told sociodemographic of Staff Nurses in a
publicly." certain private hospital.
B. "This discussion should only be
confined within this group of people. Any 36 As a nurse, you know the
information discussed should not be told appropriate data gathering tool, this is?
publicly." A. Survey
C "This discussion should not be confined B. Questionnaire
within this group of people. Any information C. Structured Discussion
discussed should not be told publicly. D. Interview Method
D. "This discussion should only be confined 37. As a nurse, these are the following
within this group of people. Any information topics in the scope and limitations except.
discussed should be told publicly."
A Limited to the hospital personnel of a similar in research where in you implement
private hospital the planned study? A. Theorize about facts
B. Limited only to the practice of and possible
preventing needle prick injuries of nursing B. Gather, analyze relevant information–
personnel C. The scope of the study C. Report findings
involves the knowledge about prevention D. Determine information needs
of needle prick injunes
D. The scope of the study involves the 42. Using the Research perspective, when
sociodemographic profile of nursing is the time that a nurse makes inferences?
Personnel A. Define purpose of review of literature
B. Formulate a problem, define variables
38. Which of the following is not a null C. Select design sample
hypothesis? D. Report findings
A. There is no relationship between
the level of knowledge and practice of 43. Which of the following is does not
prevention of needle prick injuries B. belong to the group?
There is no relationship between the A. Assess; Identify the problem
practice of prevention of needle prick B. Plan: Theorize
injuries and sociodemographic profile C. Intervene: Gather relevant information
of staff nurses D. Evaluate Outcomes
C. There is a significant relationship
between the knowledge and 44. In selecting a problem, these are the
sociodemographic profile of staff nurses in following consideration except:
a certain private hospital. D. All of the A. Time Factor
above. B. Talents
E. None of the above C. Cost
D. Data availability
39. Which of the following would
correspond to Intervene in Nursing 45. These are the purpose of Review
Process? of Related Literature except: A.
A. Select design plan Reveal investigations
B. Report findings B. Reveal sources of data
C. Implement planned study C. Reveal what is the problem– D.
D. Select a plan analysis Reveal the significant research
personalities
40. Which of the following would
correspond to Plan in Nursing Process? 46. As a nurse, you know the following a
A. Select design plan function of theoretical framework, except:
B. Report findings A. Specifies relationship among the
C. implement planned study D. concepts
Change the objectives after B. Give a graphic view of the data
implementation C. Clarifies the concept on which the study
41. When it comes to steps in Problem is built
Solving. Which of the following would be
D. State assumptions 51. in planning care for a newly admitted
patient with depression, the highest priority
47. Which of the following would not be for the nurse is
Qualitative Research? A. orienting the patient to the unit.
A. Case study of Myocardial Infarction B. encouraginq expression of feelings.
B. A Review on Corona Trial C. providing a safe environment D. meeting
C. A Review on the Nursing Uniform the patient at an appropriato affective
D. A Review on the Performance in level.
Related Learning Experience of student
nurses 52. A patient displays disorganized, difficult
to understand speech, behavioral
48. Using statistics in nursing research is a disorganization, and a silly, inappropriate
very vital tool in presenting the data. As a affect The patient prefers to sit alone and
nurse you know the definition of sampling be uninvolved in unit activities, and often
as: appears to be listening and responding to
A. Taking certain areas of the population unseen stimuli. The nursing diagnosis that
dividing the areas into sections B. should be given priority is:
Process of selecting a portion of the A. impaired verbal communication***
population to represent the entire B. social isolation.
population C. ineffective coping.
C. Using every Nth name from the list D. impaired social interaction.
of participants
D. Taking any sample as long as it 53. The nurse should focus assessment for
comes up with the quota a patient with type 1 schizophrenia primarily
on gathering data about:
49. A researcher plans to conduct a survey A. communication difficulties.
It the population on Marupok City is 67, 666, B. perceptual alterations.
find the sample size it the margin of error is C. social interactions.
23%. D. avolition.
A. 19
B. 15 54. A withdrawn patient exhibits peculiar
C. 23 gestures and waxy flexibility. She repeats
D. 29 what the nurse says to her in a high-pitched
voice but does not otherwise respond
50. The nurse develops the following verbally. The nurse should document that
hypothesis: Elderly women receive less the patient demonstrates: A. echolalia.
aggressive treatment for breast cancer than B. alogia
do younger women. Which variable would C. concrete thinking.
be considered to be the dependent D. associative looseness.
variable? 55. The emergency room phones the
A. Degree of treatment received psychiatric unit to say that a patient
B. Age of the patient demonstrating symptoms of acute
C. Type of cancer being treated schizophrenia, including altered
D. Use of inpatient treatment perceptions, is being admitted. The nurse
can anticipate care by considering that the C. helping the patient develop a
type of perceptual alteration most realistic self- concept.
commonly displayed by patients with D. identifying and supporting patient
schizophrenia is: A. waxy flexibility. strengths
B. auditory hallucinations.
C. inappropriate affect. 59. The nurse notes that a patient with
D. loose associations. obsessive- compulsive disorder (OCD) is
pacing up and down the corridor while
56. A patient relates the following history Ile counting each circuit. The best action on the
Plebence of continuous, intrusive thought part of the nurse Would be to: A. ask her
her house is contaminated with lethal why she is pacing and cousin B. take her
bacteria, and the uncontrollable urge to by the arm and lead her to her room
continuously clean the walls, Moors, and C. offer to play cards with her in the
furniture. These symptoms are most dayroom
consistent with the DSM-/V-TR diagnosis ot D. permit her to pace and count until
A. social phobia. she's comfortable
B. panic disorder.
C. somatoform disorder 60. A principle that should be applied when
D. obsessive-compulsive disorder providing care for a patient with conversion
disorder is
57. A patient whose husband was killed in A. give attention to the patient, not the
the World Trade Center explosion has symptom
become unwilling to enter a tall building B. structure care to provide time for
because she experiences severe physical rituals C. facilitate progressive review of the
and emotional symptoms when she does, trauma.
During these episodes, she becomes D. permit dependence while the symplo is
diaphoretic, her heart races, and she feels present.
as though she cannot breathe. She is filled
with dread, thinking that the building will 61. Which symptom related to
explode The nurse can assess these disordered communication is the nurse
symptoms as being most consistent with the most likely to assess in a patient who is
diagnosis of having a manic episode? A. Mutism
A. obsessive-compulsive disorder B. Flight of ideas
B. generalized anxiety disorder C. Loose associations
C. acute stress disorder. D. echolalia
D. specific phobia. 62. A principle of greatest value when
interacting with a patient who is
58. When working with a patient who has experiencing a manic episode is:
dissociative amnesia, the nurse should plan A. use a calm matter of fact approach
to begin by B. avoid mentioning limits
A. taking measures to prevent identity C. do not interrupt patient
diffusion D. encourage joking
B. setting mutual goals for behavioral
changes
63. A nursing diagnosis that can be 67. A nursing diagnosis appropriate to
established for a grossly hyperactive manic consider for a patient with any of the
patients who runs wherever he goes, personality disorders is: A. noncompliance.
exercises wildly, and is argumentative with B. impaired social interaction.
other patients is : C. disturbed personal identity.
A. powerlessness D. disturbed sensory perception.
B. risk for injury
C. deficient diversional activity 68. A patient who is suspicious of the
D. disturbed thought processes motives of others has had a long-time feud
with his two siblings over their parents' wills.
64. During community meeting, a manic He reacts quickly with anger whenever he
patient tells another patient, You need to thinks that someone is threatening his
push yourself away from the table more. welfare. The nurse would assess these
You're too fat for your own good!" The characteristics as being most consistent
nurse should intervene by: with the profile known as:
A. calmly telling the patient that unit A. schizoid personality.
rules do not permit insulting others, B. paranoid personality .
B. telling the patient that he must leave C. borderline personality.
the meeting and go to his room. D. narcissistic personality.
C. telling the patient that he can remain
in the meeting only if he apologizes. 69. The characteristic of individuals with
D. suggesting that the patient take prn dramatic erratic personality disorders that
medication. makes it advisable for staff to have frequent
65. A patient with bipolar I disorder is patient- centered meetings is the individuals
noted to be laughing and giddy one minute propensity for :
and within seconds is angry and sarcastic. A. Behaving responsibly in the peer grou
The assessment that the nurse should B. Quickly and successfully adapting to
make is that the patient's mood is: A. stress
incongruent. C. manipulating others to evade limits D.
B. inappropriate. coping successfully with a stressful
C. incandescent. Environment.
D. labile.
70. A 27-year-old is admitted for
66. A psychiatric technician mentions to diagnostic Workup. She is described as
the nurse, "I think I heard the ED doctor having a history of frequent intoxication
say that the patient I just brought to the unit and promiscuity. Recently, she has stolen
has a personality disorder." To follow up on money from her grandmother to finance a
this, the nurse could look at the diagnostic trip to Las Vegas with her new boyfriend.
sheet under the DSM-IV-TR axis She indicates that she is not sorry for
A. I stealing the money. Only sorry that her
B. II grandmother pressed charges instead of
C. III "being a good sport." The nurse assesses
D. IV this as being consistent with:
A. conduct disorder
B. narcissistic personality disorder.??? guilt and shame over molesting a child. He
C. antisocial personality disorder. is concerned about the impact on his family
D. Borderline personality disorder. and states that the family would be better
off without him. The nurse should: A.
71. The wife of a patient with a sexual explore his feelings in greater depth.
disorder asks, "What's a paraphilia?" The B. set limits on patient disclosure
nurse, who knows that the physician has C. consider instituting suicide
talked to the patient and spouse and has precaution
used this term, assumes that the spouse is D. provide prn anxiolytic medication.
seeking information. The nurse should
respond: 75. A 56-year-old man has been feeling
A. "Any homosexual act: much tension since losing his job. He
B. "Inhibition of the sexual response cycle leaves home one morning and, while sitting
C . "Intense sexual urges with an in the park feeding birds, impulsively
abnormal focus. exposes himself publicly to a group of
D. "Discomfort with one's biological mothers and children. This behavior should
gender." be assessed as:
A. voyeurism.
72. When working with a patient B. dyspareunia.
regarding sexual concerns, a necessity C. exhibitionism.
for providing nonjudgmental care is: A. D. sexual masochism.
limit setting.
B. assertiveness training. 76. When assessing a patient with
C. sexual self-awareness. cognitive disorder, the nurse should base
D. effective communication observations on knowledge that the
foundation of the cognitive process is:
73. A patient with premature ejaculation A. memory
tells the nurse, "I feel like such a failure. It's B. reasoning.
so awful for both me and my wife. Do you C. orientation,
have any suggestions that would be D. perception.
helpful?" The remark that clarifies the
nurse's role is: 77. The highest priority for nursing care
A. "Sex therapy isn't my specialty, but for a patient with dementia is A.
I will try to help. improving cognition.
B. "I can refer you to the sexual B. individualizing care .
disorders clinic to see a physician who C. maintaining an optimal level of function
specializes in this disorder" C. "Have you D. promoting self-confidence and self-
spoken with your physician about using esteem.
Viagra?"
D. "There are several techniques here in 78. An elderly individual is having difficulty
this pamphlet that might be helpful." recognizing ordinary objects such as pencils
and water faucets. When he fails to think of
74. A patient who is a pedophile tells the the word, he describes the function of the
nurse that he is feeling a huge amount of
Object- "that thing that writes" or "the thing 82. When a patient with anorexia
that gives water." The nurse assesses this nervosa spills milk over her plate of
as partially eaten food, the best approach
A. apraxia. for the nurse to take would be to say A.
B. aphasia. "Nice try, but it won't work." B Why are
C. agnosia. you deliberately making mealtime
D. amnesia. difficult?
C. "I'll get you another plate of food so
79. The nurse notes that an elderly patient you can finish."
has fluctuating levels of awareness. She D. That little trick will cost you television
seems anxious. She tells the nurse that she privileges.
saw her granddaughter standing at the foot
of the bed during the night. Later, the nurse 83. A personality characteristic the
sees her moving her hands as though nurse would expect to find in a patient
picking things out of the air. The nurse with an eating disorder is: A.
should suspect: extroversion.
A delirium. B. high self-esteem.
B. dementia. C. perfectionism.
C bipolar disorder. D. callous disregard for other
D. schizophrenia.??
84. School nurses should be
80. The patient need that is of particularly vigilant for signs of eating
primary importance in the care of a disorders: A. in fourth graders.
patient with severe Alzheimer's B. among unpopular, studious high
disease is: A. promotion of self-care school boys
activities. B. maintenance of nutrition C. among popular, high-achieving high
and hydration school girls.
C. demands that exceed capacity to D. at transitions from elementary to
function. middle school and middle school to high
D. periodic change of routine and school.
environment.
85. The nurse caring for individuals with
81. A patient with anorexia nervosa has the eating disorders should determine that the
nursing diagnosis "imbalanced nutrition: general way in which bulimic and anorexic
less than body requirements related to individuals differ is:
inadequate food intake." The expected A. indiscernible
outcome should be that the patient will: B. anorexia is life-threatening; bulimia is not
A. gain 1 to 3 pounds weekly B. C. bulimia has a biologic origin; anorexia
exhibit fewer signs of malnutrition. does not.
C. restore healthy eating patterns and D. patients with anorexia are proud of their
normalize weight. eating habits; patients with bulimia are
D. identify cognitive distortions about ashamed.
weight and shape.
86. A patient asks the nurse, "How would I 90. nurse assessing an individual who is a
know if I were dependent on alcohol?" The multidrug abuser should ask about recent
nurse should respond by telling the patient use of the drug that produces the most
that dependence is defined by: sustained high, which is:
A. a compulsion to use the drug A. crack
B. a loss of control over use of the drug. B. heroin.
C. a physiologic need to use the drug. D. C. cocaine.
continued use despite adverse D. methamphetamine
consequences.
91. An important ECT pretreatment
87. The wife of an individual who is responsibility of the nurse is to: A.
alcohol- dependent asks the nurse, "What thoroughly assess the patient's
do you mean when you ask if my husband pretreatment level of functioning.
ever experienced a blackout?" The best B. order and interpret skull and spine film
explanation would be that "A blackout is: C. obtain informed consent for treatment.
A. a comatose period related to alcohol D. prepare the light box for early morning
withdrawal." use
B. a comatose period related to alcohol
intoxication." 92. Which drug is the nurse most likely
C. a time period in which the person is To administer 30 to 45 minutes before
'passed out." ECT?
D. a period of time in which the person A. Anectine
under the influence of alcohol functions B. Atropine
normally but later is unable to remember. C. Alprazolam
D. Amobarbital
88. The nurse would suspect a disulfiram
(Antabuse)-alcohol reaction when a patient 93. An 8-year-old boy is referred to the
presents with symptoms of: clinic for diagnosis and treatment. He is
A. skin rash, itching, and urticaria. described by his parents as disobedient
B. pallor, hypotension, and muscle and argumentative. He has an explosive
cramping. temper and low frustration tolerance. Other
C. dry skin bradycardia fatigue, and children relate poorly to him. These
headache behaviors are most consistent with the
D. headache dyspnea, nausea, vomiting, medical diagnosis of:
and flushing A. autistic disorder.
B. conduct disorder.
89. A patient report experiencing insomnia C. oppositional defiant disorder.
and taking diazepam and wine in increasing D. attention-deficit/hyperactivity disorder.
amounts to be able to sleep. The nurse
should teach the patient about: A. the 94. The behaviors the nurse would expect
danger of ENS depression (CNS) to document in a child who has Tourette's
B. the risk of acetaldehyde toxicity syndrome are:
C. the risk of fetal alcohol syndrome D. A. inattention and restlessness.
diazepam and dietary precautions B. hostility and defiance.
C. body rocking and head banging. C. encourage the patient to take
D. involuntary motor movement and independent action.
vocalization. D. provide referrals contingent on the
patient leaving the abuser.
95. A nursing diagnosis that would be
universally applicable for children with 98. A survivor of childhood abuse enters
autism is: the hospital following an episode of wrist
A. impaired social interaction related to cutting. The patient has a history of binge-
inability to relate to others. purge eating disorder and difficulty trusting
B. anxiety related to disturbed thought and relauna to others. A nursing
processes intervention of high priority is:
C. chronic low self-esteem related to A. setting limits on self-harmful behavior.
excessive negative feedback. B. fostering belief in and valuing family C.
D. deficient fluid volume related to encouraging sharing of blame for the
peculiar eating habits. abuse with the perpetrator.
D. identifying and confronting abnormal
96. The nurse working with a victim of responses and feelings
spousal abuse should factor into care
planning the fact that a deterrent to 99. The nurse caring for victims of
leaving the abusive situation is the violence should plan care based on the
events that occur in the: A. tension- understanding that most victims begin to
building sage react emotionally to the effect that the crime
B. honeymoon stage has on their lives. This stage of recovery is
C. battering incident. known as: A. impact
D. processing stage B.recoil
C.reorganization
97. A woman who has been repeatedly D.retribution
abused by her partner comes to the ED for
treatment of severe contusions. As her 100. In preparing a care plan for an
injuries are being treated, she tells the abuse victim, it is most important to include
nurse "He almost killed me this time, I a long-term outcome that addresses the
quess I'm going to have to do something. need for:
The priority intervention for the nurse is to: A. moving the individual from victim to
A. tell the abuser to stop the abuse and survivor status.
seek help. B. providing support groups for long-term
B. Help the patient devise a safety or assistance.
escape plan. C. using empathy to establish rapport and
build trust.
D. shifting blame for the incident from patient to perpetrator.

You might also like